TestPrep (P) - Trauma

Ace your homework & exams now with Quizwiz!

Using the adult Rule of Nines, the anterior thorax accounts for what percent of the total body surface area? A) 9 B) 18 C) 27 D) 36

A) 9 Feedback : According to the adult Rule of Nines, the anterior trunk (thorax and abdomen) accounts for 18% of the total body surface area (TBSA). Therefore, the anterior thorax (half of the trunk} would account for 9% of the TBSA.

Which of the following findings is an indicator of decompensated shock in a trauma patient? A) Absent radial pulses B) Pale, cool, clammy skin C) Pulse rate of 120 beats/min D) Respiratory rate of 24 breaths/min

A) Absent radial pulses Feedback: Decompensated shock is generally marked by a drop in systolic blood pressure, usually below 90 mm Hg. As the systolic BP begins to fall, peripheral pulses (ie, radial) become weak and eventually absent. Pallor, diaphoresis, tachycardia, and tachypnea would also be expected in the patient with shock; however, an absence of peripheral pulses would be the most consistent with decompensated shock.

Which of the following organs would produce the MOST severe blood loss following trauma to the abdomen? A) Liver B) Spleen C) Kidney D) Pancreas

A) Liver Feedback: The liver is a very large, highly vascular organ that contains a large volume of blood at any given time. It produces several blood coagulation factors, including fibri nogen (1), prothrombin (11), ionized calcium (IV), labile factor/proaccelerin (V), VI , and stable factor/proconvertin (VII). If the liver is damaged or diseased, it can take longer for the body to fonn clots. This would result in increased bleeding time and a large volume of blood loss.

What three factors have been identified as major contributors to death in patients with severe traumatic hemorrhage? A) Acidosis, hypothermia, coagulopathy B) Alkalosis, hyperthermia, hyperkalemia C) Hypercalcemia, infect ion, hemostasis D) Hyponatremia, alkalosis, polycythemia

A) Acidosis, hypothermia, coagulopathy Feedback : The tra uma triad of death, which includes hypothermia, coagulopathy (poor blood clotting), and acidosis, is a major contributor to death in patients with severe traumatic hemorrhage. It has been well documented that even mildly hypothermic patients have a lower survival rate than normothermic patients. In addition, hypothermia contributes to coagulopathy, which also reduces survival from traumatic hemorrhage; if the blood cannot clot, the body cannot stop the bleeding (hemostasis). Any factor that interferes with blood clotting will cause the patient more blood loss than would otherwise occur. This will lead to poor perfusion and ultimately, death. Finally, acidosis (defined as blood pH of less than 7.35) often occurs with evere hemorrhage and treatments to compensate for it. For exa mple, normal saline is acidic and can increase acidosis. /\cido is contributes to coagulopathy a nd complicates treatment of the patient. In addition to the trauma lethal triad, hypocalcemia has also been identified as a contributor to death; this would form the trauma lethal diamond. It has been well established that hemorrhaging patients become hypocalcemic. Furthermore, whole blood transfusions can exacerbate existing hypocalcemia because of the preservative in the transfused blood. For this reason, it is becoming common to administer calcium following whole blood transfusion, especially multiple transfusions. Polycythemia is an overproduction of red blood cells; this would increase the oxygen-carrying capacity of the blood and is not associated with increased mortality in patients with severe traumatic hemorrhage.

What initial injury does the patient typically experience during an explosion? A) Barotrauma B) Blunt trauma C) Impaled object D) Widespread burns

A) Barotrauma Feedback: The blast from an explosion causes a wave of pressure. This wave causes the initial trauma to the patient, usually in the form of barotrauma, in which case hollow organs and the tympanic membrane (eardrum) can rupture. Burns are also common during the blast phase. Secondary injuries occur when the patient is struck by flying debris, and tertiary injuries result from the patient being thrown into fixed structures or other hard surfaces.

Mastoid bruising and cerebrospinal otorrhea following head trauma indicate which of the following? A) Basilar skull fracture B) Massive cerebral edema C) Herniation of the brainstem D) Fractured cribriform plate

A) Basilar skull fracture Feedback : Mastoid bruising (Battle sign) and cerebrospinal otorrhea (cerebrospinal fluid [CSF] leakage from the ears) are classic signs of a basilar skull fracture. Basilar skull fractures are associated with high-energy trauma, and usually occur following diffuse impact to the head (eg, falls, motor vehicle crashes). /I basilar skull fracture generally results from extension of a linear fracture to the base of the skull. Cerebrospinal rhinorrhea (CSF draining from the nose) suggests a fractured cribriform plate. The cribriform plate of the ethmoid bone is a horizontal bone that is perforated with numerous foramina (openings) that allow pas age of the olfactory nerve filament from the nasal cavity. Cerebral edema (swelling of the brain) can result from any ignificant head injury; it is not exclusive to a basilar skull fracture. Signs of brainstem herniation-which represent significant imracranial pressure-include abnorn1al respiratory patterns (ie, Biot re pirations, ataxic respirations), posturing (flexor or extensor). and pupillary abnormalities (asymmetric or fixed and dilated pupils).

A man has an open fracture of the forearm in which the bone is protruding from the wound. What should you do? A) Brush away any obvious debris from the skin surface and apply a dressing B) Gently irrigate the wound with saline or water for 30 seconds and then splint it C) Remove any large debris from within the wound and then apply a sterile dressing D) Apply gentle traction until the bone retracts into the wound and then splint it

A) Brush away any obvious debris from the skin surface and apply a dressing Feedback: As with any open injury, your priority in caring for an open fracture is to control bleeding. Then, you should gently brush away any obvious debris from the skin surface surrounding the wound; apply dry, sterile dressings; and splint the extremity appropriately. Do not irrigate the wound ; this may force outside contaminants into the wound, increasing the risk of infection. Entering or probing the open fracture site in an attempt to retrieve any debris also increases the risk of infection. Never attempt to replace exposed bone ends back into the wound; this significantly increases the ri k of infection.

During your assessment of a patient with a closed head inj ury, you note pink fluid draining from the nose. What should you suspect? A) Cribriform plate fracture B) Basilar skull fracture C) Orbital skull fracture D) Trauma to the sinuses

A) Cribriform plate fracture Feedback: Bleeding or other fluid drainage from the nose following head trauma is indicative or a fracture or the cribriform plate. The cribriform plate, an extension of the ethmoid bone, is essentially the floor or the cranial vault. When a fracture to this plate occurs, cerebrospinal fluid (CSF) may leak into the sinuses and drain from the nose (cerebrospinal rhinorrhea). CSF drainage from the ears (cerebrospinal ocorrhea) is indicative or a basilar skull fracture. Orbital skull fractures are not associated with CSF drainage from the ears or nose. Trauma to the sinuses would likely cause frank blood drainage from the nose (epistaxis).

Which of the following signs is LEAST suggestive of an underlying arterial injury in a patient with a closed extremity fracture? A) Disproportionate pain B) Weak or absent distal pulse C) Pulsatile expanding hematoma D) A palpable thrill over the i njury

A) Disproportionate pain Feedback: An arterial i njury may occur in conjunction with an open or closed fracture. In patients with a closed fracture, signs of an underlying arterial injury include a diminished or absent distal pulse, a pulsatile expanding hematoma (indicates rapid internal bleeding from an artery), and a palpable thrill (vibration) over the injury site that correlates with the patient's pulse. Searing or burning pain that is out of proportion to the injury is a common early sign of compartment syndrome - a condition associated with crush injuries, fractures, or dislocations - in which pressure builds within the osteofascial compartment (the space between groups of muscles surrounded by fascia) of an extremity.

After being struck in the head with a baseball bat, a 9-year-old boy immediately loses consciousness. On your arrival, he is conscious but confused. Shortly into your assessment, he becomes unresponsive. What should you suspect? A) Epidural hemorrhage B) Subdural hemorrhage C) Cerebral contusion D) Severe concussion

A) Epidural hemorrhage Feedback: An epidural hemorrhage, which is usually the result of damage to the middle meningeal artery, produces a loss of consciousness immediately after impact, after which the patient often has a brief return of consciousness. As the arterial bleeding begins to increase the pressure within the cranium, however, the patient's mental status rapidly deteriorates. Subdural hemorrhages are usually venous in nature; signs and symptoms may not present for up to 24 hours or longer.

For which of the following injuries would immediate transport provide the patient with the best chance of survival? A) Hemothorax B) Limb amp utation C) Sucking chest wound D) Tension pneumothorax

A) Hemothorax Feedback: Prompt transport is indicated for patients with any of the injuries listed. However, to answer this item correctly, you must understand what you can address in the field versus what you cannot. A tourniquet can be applied to a l imb amputation to stop the bleeding. An occlusive dressing can be applied to a sucking chest wound to improve ventilation. A needle decompression can be performed to resolve a tension pneumothorax. A hemothorax, however, cannot be resolved in the field because the patient has ongoing, uncontrolled hemorrhage in the thoracic cavity. Immediate transport (as in absolute minimal on scene time), with resuscitation en route, will afford these patients the best chance of urvival.

A 68-year-old woman was involved in a motor vehicle collision. She is conscious and alert and complains of pain to her shoulders and h ips. Her BP is 100/52 mm Hg, pulse is 78 beats/min, and respirations are 14 breaths/min. Assuming all levels of trauma center are within your jurisdiction, what level of trauma center should she be transported to? A) I B) II C) III D) IV

A) I Feedback : In 2022, the American College of Surgeons Committee on Trauma (ACS-COT) published an updated Field Triage Decision Scheme designed to ensure that trauma patients are transported to the most appropriate facility. Physiologic criteria for preferential transport to the highest level of trauma care within the defined trauma system consists of the patient's Glasgow Coma Scale (GCS) score, sy tolic BP, and respiratory rate. A GCS of 6 or less warrants transport to the highest level of trauma care. A systolic BP of less than 70 mm Hg (0 co 9 years of age), less than 90 mm Hg (10 to 64 years of age), or less than 110 mm Hg (65 years of age and older) warrants transport to the highest level of trauma care. A respiratory rate of less than 10 breaths/min or greater than 29 breaths/min, or the need for ventilatory support, warrants transport to the highest level of trauma care. A level I trauma center can provide total care for every aspect of injury, from prevention through rehabi litation. Because the patient is older than 65 years of age and her sy colic BP is less than 1 1 0 mm Hg, the level I facility is where she should be transported to.

Following a motorcycle accident, a man presents with deformity to the fifth and sixth thoracic vertebrae, a BP of 74/52 mm Hg, and a pulse rate of 74 beats/min. His skin is pink, warm, and dry. What should you suspect? A) Impairment of the sympathetic nervous system B) Ongoing blood loss within the thoracic cavity C) Insult to the parasympathetic nervous system D) Spinal fracture with associated myocardial damage

A) Impairment of the sympathetic nervous system Feedback: Hypotension and a normal or slow heart rate, especially with accompanying trauma to the spine, are indicative of neurogenic shock. Neurogenic shock can be caused by a number of conditions; spinal cord injury is a common cause. Because the sympathetic nerves originate from the thoracic spine, injury to this area may block the nerve pathways and inhibit the release of epinephrine and norepinephrine, which produces the typical tachycardia, pallor, and diaphoresis seen in other types of shock ( ie , hypovolemic, septic, etc).

A man with blunt chest trauma presents with labored breathing, engorged jugular veins, absent breath sounds over the l eft side of the chest, and hypotension. Which of the following describes the pathophysiology of this patient's injury? A) Increased pressure in the pleural space is compressing the great vessels. B) Blood i filling the pericardia I sac and is restricting cardiac refilling. C) The aorta ha been injured and blood is rapidly filling the thoracic cavity. D) Blood is filling the pleural space and is collapsing the lung on the left side.

A) Increased pressure in the pleural space is compressing the great vessels. Feedback : The injury described is a tension pneumothorax. As air fills the pleural space, it collapses the lung on the injured side. Untreated, this pleural tension will shift across the mediastinum. Think of the heart as an organ that is suspended in place by the vena cavae. As pleural tension shifts across the mediastinum, it kinks the vena cavae; as a result, blood cannot return to the heart. This increases central venous pressure, resulting in jugular venou di tention. If blood cannot return to the heart, cardiac output falls; as a result, the patient become hypotensive. The patient's symptoms are similar to a pericardia) tamponade; however, the unilaterally absent breath sou nds make a tension pneumothorax more li kely. If the aorta was torn and the patient was losing massive amounts of blood. you would expect the jugular veins to collapse. Blood in the pleural space (hemothorax) would also be expected to cause flat jugular veins.

Which of the following statements regarding compression injuries of the chest is correct? A) Injury tolerance to compression decreases as velocity increases. B) It takes greater than 50% compression of the chest to cause injury. C) Rib fractures occur when the chest is compressed by 5% to 10%. D) The chest wall can sustain up to 30% compression without injury.

A) Injury tolerance to compression decreases as velocity increases. Feedback: In a healthy ad ult. the chest has a compression tolerance of 20%, which means that the chest can be compressed by up to 20% from its original diameter without injury. Twenty- to thirty-percent compression of the chest typically results primarily in skeletal injuries (ie, rib fractures). Blunt trauma that causes greater than 30% compression of the chest will likely produce injury to the intrathoracic organs. When velocity increases, however, the chest wall becomes much less tolerant to compression. Lethal intrathoracic injuries can occur following a high-velocity impact that causes as little as 15% to 20% compression of the chest.

Which of the following conditions can reduce an older patient's ability to compensate physiologically during traumatic shock? A) Kyphosis B) Presbycusis C) Osteophytosis D) Spinal stenosis

A) Kyphosis Feedback : Kyphosis, abnormal curvat ure of the spinal column leading to an anteroposterior hump, often leads to impaired biomechanics, but can cause ventilatory difficulty even in the absence of trauma. Therefore, when an older patient is injured and in shock, the kyphotic spine may further inhibit respiratory function, thereby reducing their ability to breathe deeper and faster. Presbycusis is a gradual decline in hearing and is usually caused by the loss of sound conduction i nto the inner ear. Presbycusis can impair the patient's ability to communicate but does not affect their ability to compensate physiologically during shock. Osteophytosis (bone spur ) can result in decreased range of motion of the spinal column and narrowing of the spinal canal; this increases the risk of spinal cord injury following even minor trauma. Spinal stenosis is a narrowing of the spinal canal; this condition increases the l ikelihood of cord compression without any actual fractu re of the spinal vertebrae. Nthough osteophytosis and spinal stenosis increase the risk of spinal cord injury, they do not affect the older patient's ability to compensate physiologically during shock.

Which of the following factors or conditions would likely impair hemostasis and increase the risk of complications from blunt trauma? A) Liver disease B) Pancreatitis C) Polycythemia D) Beta blocker use

A) Liver disease Feedback : Hemostasis is the body's natural response to an injury. The hemostatic process involves vasoconstriction, formation of a platelet plug, coagulation, and the growth of fibrous tissue that permanently seals the damaged blood vessel. Numerous factors or conditions can interfere with the blood's abi lity to clot (the clotting cascade), thereby impairing the process of hemostasis. Hemophilia, a genetic disease that impairs the body's ability to control coagulation, would clearly impair hemostasis. Liver dysfunction or disease (ie, cirrhosis, cancer) would also impair hemostasis. The liver produces many of the body's coagulation factors. Medications such as warfarin sodium (Coumadin) and aspirin both impair hemostasis. Coumadin suppresses the liver's ability to produce certain clotting factors, and aspirin decreases the ability of the platelets to agg regate (clump toged1er). Other medications that can lead to continued bleeding include ticlodipine (Ticlid) and clopidogrel bisulfate (Plavix). Nthough beta blockers do not have an affect on hemostasis, they can affect the ability of a person to compensate for blood loss. By suppressing the sympathetic nervous system, beta blockers may blunt the compensatory responses of tachycardia and increa ed cardiac contractility. Pancreatitis would likely result in blood glucose derangements due to insufficiencies of glucagon and in ulin. Polycythemia-an overproduction of red blood cel ls-is a common finding in patients with emphysema ( hence the term, " pink puffer"); this condition would not affect the process of hemostasis. Patients with impaired hemostasis can die from minor internal bleeding that an otherwise healthy person would survive.

A hiker fell 25 feet and is semiconscious. There is obvious deformity to his thoracic spine and he has a large laceration on his forehead. His BP is 60/40 mm Hg, pulse is 60 beats/min, and respirations are 26 breaths/min. His face and chest are pale and diaphoretic, but his abdomen and lower extremities are pink, warm, and dry. Which of the following BEST describes the pathophysiology of these findings? A) Loss of nervous system control over the systemic vasculature B) Severe bleeding into the thoracic cavity from a ruptured aorta C) Systemic vasoconstriction d ue to nervous system hyperactivity D) Increased intracranial pressure due to bleeding within the brain

A) Loss of nervous system control over the systemic vasculature Feedback : The patient's clinical presentation indicates neurogenic shock. With this type of shock, nervous system control over the vasculature is lost and catecholamines (epinephrine, norepinephrine) are not released. /Is a result, the systemic vasculature dilates, resulting in hypotension. /Is with other types of hock (ie, hypovolemic, septic, anaphylactic), the patient with neurogenic shock is not tachycardic; they are either nom1ocardic or bradycardic. In a spinal injury with neurogenic shock, the skin above the level of the injury is often pale and cool, whereas the skin below the level of the injury is warm and dry. This patient's signs and symptoms are less consistent with internal hemorrhage. in which case you would expect him to be tachycardic. I ncreased intracranial pressure is also less likely; such patients typically present with hypertension and bradycardia.

A patient with blunt abdominal trauma has a BP of 60/40 mm Hg, a heart rate of 120 beats/min , and a respiratory rate of 22 breaths/min. Intravenous fl uids should be given to target which of the following? A) MAP of 60 to 65 mm Hg B) Systolic BP of 70 mm Hg C) MAP of 80 to 85 mm Hg D) Systolic BP of 100 mm Hg

A) MAP of 60 to 65 mm Hg Feedback : The decision to administer crystalloid fluid replacement depends on the patient's clinical presentation. Abdominal trauma represents one of the key situations in which balanced resuscitation is indicated. Aggressive IV fluid administration may elevate the patient's BP to levels that will disrupt any clot that has formed and result in recurrence of bleeding d1at has ceased because of hemostasis. Whether crystalloid or blood products are available, paramedics must achieve a delicate balance: maintain a BP that provides perfusion to vital organs without restoring the BP to elevated or even normal ranges. I ncreasing the BP too much may reinitiate bleeding within the abdomen or pelvis. In the ab ence of a traumatic bra i n i njury, the target systolic BP is 80 to 90 mm Hg, or a mean arterial pressure (MAP) of 60 to 65 mm Hg. If the patient's baseline BP and/or MAP are already at these levels, crystalloid fluids should not be given.

Which of the following injury mechanisms and clinical findings would MOST likely warrant transport to a facility that provides the highest level of t rauma care? A) Motorcycle crash; pelvic instability; systolic BP of 100 mm Hg B) Fall from a standing position; no loss of consciousness; GCS of 14 C) Ro llerblade accident ; h umeral fracture; heart rate of l00 beats/min D) Small-caliber gunshot wound to the calf; heart rate of 110 beats/min

A) Motorcycle crash; pelvic instability; systolic BP of 100 mm Hg Feedback: According to the 2011 CDC guidelines for the field triage of injured patients, the following injury mechanisms or clinical findi ngs warrant transport to highest level of care in a defined trauma system: Glasgow Coma Scale less than or equal to 13; systolic BP less than 90 mm Hg; respiratory rate less than 10 or greater than 29 breaths/min (or the need for ventilatory support); all penetrating injuries to the head, neck, torso, and extremities proximal to the elbow or knee; chest wall instability or deformity (eg, flail chest); two or more proximal long bone fractures; crushed, degloved, mangled, or pulseless extremity; amputation proximal to the wrist or ankle; pelvic Fractures; open or depressed skull fractures; and paralysis. The patient who crashed his motorcycle is the only patient who meets the trauma triage c1iteria.

Following a significant electrical burn, a patient begins passing tea-colored urine. What is responsible for this? A) Myoglobin B) Potassium C) Methemoglobin D) Carboxyhemoglobin

A) Myoglobin Feedback : Muscle necrosis following an electrical burn releases myoglobin, a molecule found in the muscle that assists the muscle tissue in the transportation of oxygen. When released into the bloodstream in considerable amounts, the myoglobin can obstruct the renal collecting tubules, leading to acute renal failure. This condition, myoglobinuria, is evidenced by tea- or cola-colored urine. Electrical burns can also cause the muscles to release potassium. A significant increase in serum potassium can cause life-threatening dysrhythmias; potassium is not responsible for dark urine. Methemoglobin i an abnormal form of hemoglobin that cannot bind to oxygen. No,mally, 1% to 2% of a patient's total hemoglobin is methemoglobin; higher percentages can be genetic or following exposure to certain chemicals. Electrical burns do not cause an increase in methemoglobi n. Carboxyhemoglobin is hemoglobin that is bound to carbon monoxide (CO) instead of oxygen. An increase in carboxyhemoglobin would occur in patients exposed to CO, such as entrapment in a structural fire; electrical burns do not result in increased CO levels.

Following a crush injury in which both lower extremities are compressed for an extended period, the release of which substances can cause metabolic acidosis, renal failure, and dysrhythmias when the compressive force is released? A) Myoglobin, potassium and lactate B) Methemoglobin, sodium, and calcium C) Troponin T, magnesium, and sodium D) Creatine phosphokinase and calcium

A) Myoglobin, potassium and lactate Feedback : Crush syndrome occurs because of a prolonged compressive force that impairs muscle metabolism and circulation and presents following the extrication or release of an entrapped limb. When limbs are crushed, muscle tissue becomes ischemic and dies. Necrosis then develops, resulting in release of harmful products, a process called rhabdomyolysis. Rhabdomyolysis is not only seen in trauma patients; it can also occur in patients who have been lying on an extremity, on their back, or in the same position for an extended period (4 to 6 hours of compres ion). Patients who have overdosed or had a stroke and are not found for an extended period can develop rhabdomyolysis. When the force compressing the region is released, blood flow is reestablished, and the necrot ic tissue is released into the systemic circulation. The primary substances of concern are lactate, potassium, and myoglobin. The release of these substances into the circulation can cause metabolic acidosis. Furthermore, potassium can cause life-threatening dysrhythmias, and myoglobin can block the renal tubules and cause renal failure. Methemoglobin is an abnormal form of hemoglobin that does not have the ability to carry oxygen; methemoglobinemia can be congen ital or acquired. Troponin I and T are cardiac biomarkers; they are proteins released when the myocardium is injured. Creatine phosphokinase (CPK) is also a cardiac biomarker used to diagnose acute myocardial infarction, myocardial ischemia, or myocarditis.

Which of the following is of LEAST pertinence when assessing the mechanism of injury of a patient who was involved in a motor vehicle crash? A) Patient's weight B) Age of the patient C) Airbag deployment D) Type of object struck

A) Patient's weight Feedback : Assessing the mechanism of injury ( MO!) allows the paramedic to predict, to some degree, the type and severity of injuries by noting certain findings pertaining to how the injury occurred. These include the type of object struck (there is a BIG difference between striking a stop sign pole and a bridge pillar), the speed of the vehicle upon impact, whether or not safety devices (ie, lap belt and shoulder harness) were in use at the time of impact, and whether or not the airbag-if equipped with one-deployed upon impact. The age of the patient is also an important factor to con ider; a 70-year-old patient, who has a greater likelihood of having an underlying medical condition (eg, osteoporosis, cardiovascular disease), is more likely to sustain serious injury from mechanisms that may not otherwise significantly injure a younger, healthier patient. You should routinely note tl1e patient's approximate weight; however, this is not as important a factor when assessing tl1e MOI as those previously mentioned.

Which of the fo llowing is an abnormal physiologic process that occurs at the capillary level during shock? A) Precapillary sphincter relaxation in response to lactic acid buildup B) Minimal capillary blood now, which results in aerobic metabolism C) Sustained pre- and postcapillary constriction in response to hypoxia D) Postcapillary sphincter relaxation that causes capillary fluid depletion

A) Precapillary sphincter relaxation in response to lactic acid buildup Feedback : As perfusion decreases, cellular ischemia occurs. Minimal blood flow passes through the capillaries, causing cellular conversion from aerobic metabolism to anaerobic metabolism, which produces lactic acid and can quickly lead to metabolic acidosis. With less circulation, blood stagnates in the capillaries. The precapil lary sphincter relaxes in response to the buildup of lactic acid, vasomotor center failure, and increased carbon dioxide levels. The postcapillary sphincters remain constricted, causing the capillaries to become engorged with fluid. The capillary sphincters - circular muscular walls that constrict and dilate - regulate blood flow through the capillary beds and are under the control of the autonomic nervous system. Among other factors (ie, heat, cold), capi l lary sphincters respond to an i ncreased demand for oxygen and the need for waste removal. Thus, regulation of blood flow is determined by cellular need and is accomplished by vascu lar constriction or dilation, working in tandem with capillary sphincter constriction or dilation.

Shortly after applying a nonventing chest seal to a woman with a sucking chest wound, her respiratory status deteriorates, and it becomes necessary to provide positivepressure ventilation. What should you do? A) Remove the chest seal and leave the wound open B) Proceed immediately with endotracheal intubation C) Perform needle decompression thro ugh the chest seal D) Ventilate her at a rate that is between 20 and 24/min

A) Remove the chest seal and leave the wound open Feedback : If a patient with a sucking chest wound is breathing adequately on their own, leave the chest seal in place, monitor for signs of pleural tension, and briefly remove the seal as needed. However, if it becomes necessary to provide positive-pressure ventilation, the chest seal should be removed; this is especially true if a nonventing chest seal was used. Covering an open pneumothorax converts it to a closed pneumothorax, and positive-pressure ventilation can quickly cause pleural tension and hemodynamic compromise. If a vented chest seal was applied, monitor it carefully to ensure that air is escaping d u ring exhalation. The vents on the chest seal can get clogged with clotted blood, dirt, and other debris; if this occurs, remove the vented chest seal before initiating positive-pressure ventilation. ft makes no sense to perform a needle decompression through a chest seal: why make a small hole when you can simply remove the chest seal and create a bigger one for air to escape from the pleural space faster? Furthermore, the catheter can get clogged with material from the chest seal. A ventilation rate of 24 breaths/min is too fast for an adult; this would only exacerbate their chest injury. Start with a rate of 1 0 breaths/min (one breath every 6 seconds) and use end-tidal CO2 to adjust your rate accord ing ly.

A woman experienced blunt abdominal trauma. When she is positioned supine, she experiences severe pain in her left shoulder. What should you suspect? A) Ruptured spleen B) Lacerated bowel C) Perforated stomach D) Injury to the kidney

A) Ruptured spleen Feedback: Kehr sign is the occurrence of acute pain in the tip of the shoulder due to the presence of blood or other irritants in the abdominal cavity when a person is lying down and the legs are elevated. Kehr sign in the left shoulder is considered a classic symptom of a ruptured spleen. Hollow organ injury, such as a lacerated bowel or perforated stomach, would be expected to cause diffuse peritoneal signs (ie, tenderness, rigidity, guarding).

A woman was struck by lightning while working in her garden. Her husband tells you d1at when he first found her, she was unconscious and not breathing. She is conscious and alert but has no recollection of the event. Her BP is 134/74 mm Hg, pulse i s 88 beats/min, and respirations are 1 8 breams/min. The cardiac monitor reveals a sinus rhythm with occasional PACs. She is moving all her extremities and has no obvious burns or musculoskeletal inj uries. What should you do? A) Start a saline lock IV B) Give 2.5 mg midazolam I M C ) Give oxygen v i a nonrebreathing mask D) Start an JV and give 1 mg/kg of lidocaine

A) Start a saline lock IV Feedback : Retrograde amnesia and an absence of bums or other trauma suggests that the patient experienced the effects of a side flash (splash effect) when lightning struck a nearby object; it is doubtful she took a direct hit. However, do not ignore her husband's report that she was unresponsive and not breathing. Brief asystole that spontaneously convert to a perfusing rhythm may occur following a lightning strike. Treatment is based on the presence of i nj u ries and the patient's hemodynamic status. In addition to a complete set of vital signs, you should obtain a 12-lead ECG to look for evidence of myocardial injury. I t would be reasonable co place an IV saline lock in case medications or fluids are needed during transport. Antidysrhythmics, such as lidocaine or amiodarone, are not indicated; she is not experiencing a life-threatening dysrhythmia. Oxygen is not indicated because there is no evidence of respiratory compromise or hypoxemia. There would be no benefit in g iving her a benzodiazepine, such a m idazolam (Ver ed) or diazepam (Valium); she is not anxious and there is no evidence of seizure activity.

Which of the following assessment findings indicates developing compartment syndrome in a patient with a lower extremity fracture? A) The pain is greater than one would expect for the injury. B) The extremity becomes increasingly warmer and pinker. C) Distal pulses are bounding and reflexes are hyperactive. D) The pain subsides during passive stretch of the extremity.

A) The pain is greater than one would expect for the injury. Feedback: Compartment syndrome occurs when excessive pressure builds up in between the m uscle and fascia (osteofascial compartment) and inhibits blood now. The syndrome can occur following a fracture, crush injury, or venomous snakebite. Signs and symptoms of compartment syndrome include pain, pallor, pulselessness, parasthesia, para lysis, and poikilothermia (cold extremity). The pain is often disproportionate to the injury and is exacerbated by passive stretch of the extremity. Prompt transport of the patient is essential. A fasciotomy is required in order to relieve pressure from the osteofascial compartment and restore circulation. This is a surgical procedure and is not performed in the prehospital setting.

Which of the following is a criterion for transporting an adult patient to a trauma center capable of providing the highest level of care? A) Two proximal long bone fractures B) GCS that is equal to or less than 14 C) Respiratory rate of 24 breaths/min D) Fall from a height of 10 to 15 feet

A) Two proximal long bone fractures Feedback : According to the 2011 Guidelines for Field Triage of Injured Patients, published by the Centers for Disease Control and Prevention (CDC), the paramedic should use certain predefined criteria when determining the most appropriate transport destination for the injured patient; these criteria are based on physiologic findings, anatomic fi ndings, mechanism of injury, and special patient considerations. According to the guidelines, two or more proximal long bone fracture is an anatomic criterion for preferential transport to the highest level of care within the defined trauma system. A Glasgow Coma Scale (GCS) score that is equal to or less than 13 or a respiratory rate less than IO breaths/min or greater than 29 breaths/min are physiologic criteria for transport to the highe t level of t rauma care. If an adult falls from greater than 20 feet, but does not meet any of the physiologic or anatomic criteria, he or she should be transported to a t rauma center; depending on the defined trauma system, this need not be the highest level trauma center. If die patient does not meet any of the predefined criteria for transport to the highest level of trauma care, the paramedic should use their j udgment or follow local protocol.

A woman who experienced a closed head injury is responsive only to pain. Her BP is 80/50 mm Hg, pulse is 120 beats/min and weak, and respi rations are 8 breaths/min and irregular. Which of the following statements regarding this patient is correct? A) You should intubate the patient and give IV fluid boluses to maintain a systolic blood pressure of at least 90 mm Hg. B) You should s uspect that the patient is experiencing neurogenic shock in addition to increased intracranial pressure. C) The patient's brain is herniating, so you should hyperventilate her with a bag-mask device at a rate of 20 breaths/min. D) Instead of giving IV fluid boluses, administer a vasopressor drug until her systolic blood pressure is at least 110 mm Hg.

A) You should intubate the patient and give IV fluid boluses to maintain a systolic blood pressure of at least 90 mm Hg. Feedback : Because of this patient's markedly decreased level of consciousness following her head injury, her airway should be definitively secured to prevent aspiration if she vomits. She will also require ventilation support. Because she is not completely unresponsive, sedation and neuromuscular blockade should be used to facilitate intubation. Once she has been intubated, ventilate her at a rate of 10 breaths/min. There is no evidence of brain herniation (eg, unresponsiveness, decerebrate posturing, asymmetric pupils). so she should be ventilated at a normal rate, while maintaining an ETCO<sub>2</sub> of 35 co 40 mm Hg. Her vital signs-hypotension and tachycardia-are not consistent with an isolated closed head injury; they are more consistent with shock, probably from an occult hemorrhage. If an adult with a seemingly isolated head injury presents with signs of shock, look for other injuries! A single episode of hypotension (<90 mm Hg) in the adult with a head injury can cause cerebral ischemia and increase mortality significantly. Give isotonic crystalloid fluid boluses as needed to maintain a systolic BP of at least 90 mm Hg. It is unlikely that the patient is in neurogenic shock, which is characterized by hypotension and a low (or relatively slow) h eart rate; she is tachycardic. which indicates that her sympathetic nervous system is intact and is releasing catecholamines. Vasopressor drugs may be useful in treating patients with neurogenic shock that is refractory co IV fluid boluses; however, this patient is not in neurogenic shock.

Common signs of a maxillofacial fracture include all of the following, EXCEPT: A) asymmetric pupils. B) dental malocclusion. C) impaired ocular movement. D) anterior or posterior epistaxis.

A) asymmetric pupils. Feedback: Unless accompanied by a traumatic brain injury and increased intracranial pressure, asymmetric (unequal) pupils are not commonly observed in patients with maxillofacial fractures. However, pat ients with orbital "blowout" fractures often complain of diplopia (double vision) and lose sensation above the eyebrow or over the cheek secondary to associated nerve damage. Fractures of the inferior orbit - the most common type - may cause paralysis of upward gaze; the patient's injured eye will not be able to follow your finger above the midline. Maxillofacial structures that may be fractured include the mandible, maxilla, orbit, and nasal bone. Crepitus, ecchymosis, instability, and swelling are common to all maxillofacial fractures. In addition, mandibular fractures often present with dental malocclusion ( misalignment of the teeth), and nasal fractures are often accompanied by anterior or posterior episcaxis (nosebleed).

A man has a closed head injury and is unresponsive. His left pupil is dilated and he has extensor posturing. His BP is 180/100 mm Hg, his pulse is 50 beats/min, and his respirations are 8 breaths/min and irregular. Treatment for him should include: A) intubation with ventilations aimed at maintaining an ETC02 of 30 to 35 mm Hg. B) high-flow oxygen via nonrebreathing mask and 10 mg of labetalol IV or 10 push. C) int ubation with ventilations at 30 breaths/min and 0.5 mg of atropine IV or 10 push. D) high-flow oxygen via nonrebreathing mask and 20 mLjkg of a crystalloid solution.

A) intubation with ventilations aimed at maintaining an ETC02 of 30 to 35 mm Hg. Feedback : The patient has a severe traumatic brain injury (TBI) with a classic Cushing triad presentation (ie, hypertension, bradycardia, irregular breathing). There is also evidence of cerebral herniation (ie, unresponsive, asymmetric pupils, extensor (decerebrate] posturing). His airway should be secured with an ET tube. The Brain Trauma Foundation (BTF) recommends ventilation at 20 breaths/min in adults if signs of cerebral herniation are present. Optimally, you should ventilate the patient to maintain an ETCO2 between 30 and 35 mm Hg. In such patients with brain injury, brief periods of increased ventilation may be beneficial. The patient's BP clearly does not warrant nuid boluses; in fact, nuid boluses would likely only worsen his intracranial pressure. Other than medications used to facilitate intubation (ie, sedation, paralytics) or treat seizures, prehospital pharmacologic therapy is usually not administered to patients with a TBI.

Common clinical signs of a hemothorax include all of the following, EXCEPT: A) mediastinal shift. B) labored tachypnea. C) pallor and tachycardia. D) collapsed jugular veins.

A) mediastinal shift. Feedback: Hemothorax - the accumulation of blood in the pleural space - is caused by bleeding from the lung parenchyma or damaged vasculature. Hemothorax manifests with signs of shock and respiratory compromise. Common clinical signs include difficulty breathing and tachypnea (labored tachypnea), pallor, diaphoresis, tachycardia, narrowing pulse pressure, hypotension, and collapsed jugular veins. In rare cases, a contralateral mediastinal shift may be observed; however, this is most reliably assessed with a chest radiograph.

Most chemical burn inj uries are treated with: A) water. B) baking soda. C) calcium gluconate. D) hydrogen peroxide .

A) water. Feedback: According to the American Burn Association, nothing has been found to be superior to water when treating most chemical burn injuries. Because many neutralizing agents produce heat and may increase inju ry severity, antidotes and neutralizing agents should generally be avoided in the prehospital setting. Alkali metals - sodium and potassium - can react violently with water and produce large amounts of heat; therefore, burns caused by these chemicals should not be irrigated with water. Depending on your local protocol, 10% calcium gluconate may be used in the prehospital setting as a neutralizing agent for hydrofluoric acid burns.

By which of the following mechanisms can patients with crush syndrome experience lethal cardiac dysrhythmias? A) Hyperkalemia B) Hyponatremia C) Myoglobinuria D) Hypercalcemia

A} Hyperkalemia Feedback : An extremity that is crushed during a traumatic injury can cause a condition called rhabdomyolysis. This condition is associated with death of the muscle in the crushed extremity and release of myoglobin. Clinically it is characterized by renal failure, end-organ injury, and, potentially, death. The timing for the impact from this myoglobin is after the crushing force is removed from the extremity. Traumatic injury to muscle causes release of both myoglobin and potassium. Once the patient has been extricated, the affected limb suddenly becomes reperfused with new blood; at the same time, the old blood with elevated levels of myoglobin and potassium is washed out of the injured area and into the rest of the body. Free myoglobin can eventually lead to renal failure. Elevated serum potassium 01yperkalemia) can result in life-threatening cardiac dysrhythmia , such as ventricular tachycardia or ventricular fibrillation. For this reason, some patients may be treated with crystalloid fluid boluses, calcium chloride, and perhaps sodium bicarbonate before they are freed, especially if they were entrapped for several hours. While hyponatremia (low serum sodium level) and hypercalcemia (high serum calcium level) can cause cardiac problems, these electrolyte abnonnalities are not typically associated with crush syndrome.

A patie nt with a severe closed head injury has bilaterally dilated and nonreactive pupils. What does this indicate? A) Injury to cranial nerves I and VII B) Pressure on the oculomotor nerve C) Severe injury to the hypothalamus D) Herniation of the upper brainstem

B) Pressure on the oculomotor nerve Feedback: The oculomotor nerve (crania l nerve 1 11) arises from the midbrain and exits the brain to each eye. It controls the upper eyelid muscle, which raises the eyelid; the extraocular muscle, which moves the eye inward; and the pupillary muscle, which constricts the pupil. Damage to or pressure on this nerve will cause the pupils to dilate and fail to constrict when light is shone into them. Common causes of this include increased intracranial pressure, stroke (both ischemic and hemorrhagic), and cerebral hypoxia. Cranial nerve I is the olfactory nerve; it controls the sense of smell. Cranial nerve VII is the facial nerve; among innervating the facial muscles, cranial nerve VI I controls eyelid movement to include closing of the eyelids. This nerve does not regulate pupillary function.

A man has partial thickness burns to his face, chest, and arms. He is conscious and alert and complains of intense pain. He is still wearing his shirt, but there are areas of smoldering fabric that have adhered to his skin. What should you do? A) Remove the part of his shirt that is not adhered to the skin and apply water to the adhered fabric to ensure the burning process has stopped B) Douse his torso with sterile water for at least 15 minutes and then carefully remove his shirt, includi ng the fabric that is adhered to his skin C) Leave his shirt on, apply copious amounts of sterile water to ensure the burning process has stopped, and apply a sterile burn sheet D) Irrigate the burned areas with sterile water or saline, administer analgesia, and then carefully pull the adhered fabric from his skin

A} Remove the part of his shirt that is not adhered to the skin and apply water to the adhered fabric to ensure the burning process has stopped Feedback : After ensuring your own safety, your next priority when caring for a burn patient is to ensure that the burning process has stopped. You should remove the patient's clothing completely while simultaneously cooling the burn with sterile water or saline; this will ensure that heat is not trapped under the clothing. Smoldering pieces of fabric that are adhered to the skin should be cut, not pulled, away from the skin. Smoldering pieces of fabric that cannot be removed from the ski n should be soaked with sterile water to ensure that the burning process has stopped. After ensuring that the burn has been appropriately cooled, apply a sterile burn sheet to the burned area(s). Establish vascular access, preferably in an unburned area, and administer analgesia (eg. fe ntanyl, morphine) as needed for pain relief.

A man with a head injury opens his eyes only when you speak to him. When you ask him his name, he replies slowly by saying " o thank you:· When you apply a painful stimulus by pinching his arm, he pulls his arm away from you. What is this patient's Glasgow Coma scale score? A) 9 B) 10 C) 11 D) 12

B) 10 Feedback : The Glasgow Coma Scale (GCS) is a valuable tool when performing neurologic assessments of a patient with a head injury. A single GCS assessment cannot reliably capture the patient's clin ical progression; therefore, you should obtain a baseline GCS score and frequently (at least every 5 minutes) reassess it. Furthermore, when relaying the patient's GCS to the emergency department physician, state where the patient's deficits are; don not simply give a number. Your patient's present GCS score is a IO. He opens his eyes o nly when you speak to him; under the "eye opening" component of the GCS, this equates to a score of 3. Using inappropriate words -stating "No thank you" when you ask him his name-equates to a score of 3 under the "verbal response· component of the GCS. Withdrawal from pain (ie, pulling his arm away when a painful stimulus is applied) equates to a score of 4 under d1e "motor response· component of the GCS. A GCS score of 13 to 15 indicates a mild traumatic brain injury (TB!), a score of 8 to 12 indicates a moderate TB!, and a score of 3 to 8 indicates a severe TB!.

Which of the following patients is in need of rapid extrication following a motor vehicle crash? A) 16-year-old girl with a laceratio n to her forehead and tachycardia B) 28-year-old man with a unilateral femur fracture and confusion C) 40-year-old man with an open head injury and exposed brain mater D) 56-year-old woman with a Calles' fracture and emotional upset

B) 28-year-old man with a unilateral femur fracture and confusion Feedback: The patient with confusion would be t he best candidate for rapid extrication from an automobile. Any altered mental status following t rauma should be assumed to be the result of head injury, cerebral hypoxia, or both. An open head injury with exposed brain mater is an ominous finding that is generally not compatible with life. Rapid extrication is also indicated if a stable patient blocks access to a more critical patient.

According to the Rule of Ten, how much IV fluid per hour should be given to a 65-kg man with 36% total body surface area burns? A) 215 mL B) 400 mL C) 505 mL D) 600 mL

B) 400 mL Feedback: In an effort to simplify the process of calculating fluid requirements for burn patients in the prehospital setting, researchers from the U.S. Army Institute of Surgical Research (USAJSR) developed the Rule of Ten to help guide initial fluid resuscitation. The percentage of total body su ,face area (TBSA) burned is calculated and rounded to the nearest 10. For example, a burn of 36% would be rounded to 40%. The percentage is then multiplied by IO to get the number of mL per hour of crystalloid. Therefore, a patient with 40% TBSA burns should receive 400 mL/hr of crystalloid. This formula is used for adults weighing 88 to 154 lb (40 to 70 kg). If the patient exceeds this weight range, for each 10 kg in body weight over 70 kg, an additional 100 mL/hr is given.

A 170- pound man has partial- and full-thickness burns to 45% of his body surface area. Transport to the closest appropriate facility will be prolonged, so medical control has ordered you to begin IV fluid therapy based on the Consensus formula. How much IV fluid should this patient receive per hour? A) 712 mL B) 866 mL C) 934 mL 0) 1,045 mL

B) 866 mL Feedback: The Consensus formula, also known as the Parkland formula, is used to determine how much IV nuid a burn patient should receive during the first 24 hours following the burn. It is calculated as follows: 4 mL , body weight (in kg) , percentage of body surface area (BSA} burned. Thus, a 170 -pound (77 kg) patient with burns to 45% of his BSA should receive 13,860 mL in the first 24 hours, as follows 4 m L , 77 (kg) , 45 (BSA burned) = 13,860 mL. The Consensus formula further states that half of the 24-hour nuid volume should be given during the first 8 hours. This would equal 6,930 mL ( 13,860 mL + 2). Therefore, the patient should receive 866 mL/hr (6,930 + 8 = 866).

A man has an open abdominal wound with a small loop of bowel protruding from the wound. He is conscious and alert and is in severe pain. His BP is 126/86 mm Hg, pulse rate is 120 beats/min, and respirations are 20 breaths/min. What should you do? A) Position him prone and transport at once B) Administer 0.5 to 1 mcg/kg of fentanyl IV C) Cover the wound with a dry, sterile dres ing D) Gently place the bowel back into the wound

B) Administer 0.5 to 1 mcg/kg of fentanyl IV Feedback : Treatment for an abdominal evisceration should focus on protecting the exposed viscera from further injury. Most of the abdominal contents require a moist environment. If the intestine or some of the other abdominal organs become dry, cell death can occur. Therefore, the eviscerated abdominal contents should be covered with a sterile dressing that has been moistened with saline. These dressings should be periodically remoistened with saline to prevent them from drying out. Wet dressing may be covered with a large dry or occlusive dressing to keep the patient warm. Do OT attempt to push the exposed contents back into the wou nd; this markedly i ncreases the risk for i nfection. Tra nsport the patient in a position of comfort (prone is NOT a comfortable position for this patient) and transport without delay. Any action that increases pressure within the abdomen, such as crying, screaming, or coughing, can force more of the abdominal contents outward. Pain control is important in these patients, and because his vital signs are stable, you should administer analgesia; fentanyl 0.5 to I mcg/kg IV would be appropriate for him.

Which of the following is the MOST significant complication of an open-book pelvic fracture? A) Laceration or rupture of the urinary bladder that causes an infection when urine spills into the pelvic cavity B) An increase in pelvic volume with more internal blood loss than pelvic fractures caused by lateral compression C) Severe damage to adjacent nerves, which can result in permanent neurologic deficit in the lower ext remities D) Inability to effectively stabilize the pelvis without causing injury to the adjacent nerves, arteries, and veins

B) An increase in pelvic volume with more internal blood loss than pelvic fractures caused by lateral compression Feedback : Anteroposterior compression of the pelvis, which can result from rapid deceleration, may cause an "open-book" pelvic fracture in which the pubic symphysis spreads apart. The subsequent increase in volume of the pelvis means a patient with internal pelvic bleeding may lose a much larger volume of blood than someone without an open-book fracture. Such patients will require IV fluid boluses with an isotonic crystalloid solution (whole blood would be better), but may still remain hypotensive in the field. Lateral compression of the pelvis results from a side impact and generally does not result in an unstable pelvis. Because the volume in the pelvis is reduced, not increased, the ri k of life-threatening hemorrhage i lower than with an open-book fracture. If you suspect an open- book pelvic fracture in a hypotensive patient, tie a sheet around the patient's hips at the level of the superior anterior iliac crests; this reduces the pelvic volume. There are several devices specific to the management of this type of pelvic injury that provide superior stabilization and are easy to apply. One device, the Sam Sling, has a patented "autostop" buckle to provide the correct circumferential force to stabilize open-book pelvic fractures. Pelvic fractures occasionally cause urinary bladder damage as a result of penetration by bony fragments. Although this complicates the clinical picture, internal hemorrhage is clearly a more significant concern.

A man who crashed his motorcycle is found lying supine about 30 feet from his bike. Bystanders removed his helmet before you arrived. He has bilaterally closed femur fractures and multiple abrasions and lacerations to his arms and legs. What should yo u do? A) Administer high-flow oxygen and perform a rapid head -to- toe as essment B) Ask your partner to stabilize the patient's head as you assess his airway C) Quickly place the patient onto a backboard and load him into the ambulance D) Carefully straighten his deformed femurs as your partner talks to the patient

B) Ask your partner to stabilize the patient's head as you assess his airway Feedback : When assessing a patient with severe trauma, do not be distracted by visually impressive injuries' It is essential to complete the primary assessment of the patient and treat what will kill them first. Unless associated with severe external bleeding, musculoskeletal injuries-as grotesque as they may be-are not going to immediately kill your patient. Based on this pat ient's mechanism of i njury, you should immediately direct you r partner to manually tabilize the patient's head while you assess airway, breathing, and circulation. I f you find an immediate life threat in any of these areas, fix it and then move on with your assessment. A rapid head- to-toe assessment should then be performed to identify and treat other critical injuries. Remember, it is the ugliest injuries that are often the least life th reatening.

You are treating a patient with penetrating chest trauma. His initial vital signs revealed a blood pressure of 100/60 mm Hg, a pulse of 120 beats/min, and respirations of 2 8 breaths/min. Which o f t h e following repeat vital signs is MOST suggestive of a pericardia! tamponade? A) BP of 80/40, pulse rate of 120 B) BP of 90/70, pulse rate of 128 C) BP of 104/70, pulse rate of l00 D) BP of 160/90, pulse rate of 90

B) BP of 90/70, pulse rate of 128 Feedback: Because of blood accumulation within the pericardia! sac, cardiac contraction is restricted, which causes a drop in the systolic blood pressure. For the same reason, complete relaxation is inhibited, which causes an increase in the diastolic blood pressure. The difference between the systolic and diastolic pressures is called the pulse pressure, which becomes narrowed with a pericardia! tamponade. A blood pressure of 90/70 mm Hg has a pulse pressure of only 20 mm Hg, which is less than any of the other values listed.

Following a blunt chest injury, a patient has paradoxical movement of the chest. His respiratory rate is 30 breaths/min and his heart rate is 110 beats/min. He is receiving high-flow oxygen via nonrebreathing mask, and his O>,')'gen saturation is 87%. What should you do? A) Cover the flail segment with a bulky dressing B) Begin treatment with CPAP at 2.5 to 5 cm/H2O C) Administer 2.5 to 5 mg midazolam IM for sedation D) Ventilate with a bag-mask device at 24 breaths/min

B) Begin treatment with CPAP at 2.5 to 5 cm/H2O Feedback : Management of a flail chest is directed toward pain relief, ventilatory support, and monitoring for deterioration. The patient's ventilatory rate may be the most important parameter to follow and carefully measure. Patients who are developing underlying pulmonary contusion and respiratory compromise will demonstrate an increase in their ventilatory rate over time. Oxygen should be administered to ensure an oxygen saturation of at least 94%. As one can imagine, the patient will be in severe pain and will avoid deep breaths to minimize the pain (respiratory splinting). This will only lead to a further reduction in tidal volume. I n addition, as the nail segment collapses du ring inhalation, it can cause alveolar collapse (atelectasis), which further impairs oxygenation. Because d1is patient's oxygen saturation is remaining low despite high-flow oxygen therapy. treatment with CPJ\P would be a reasonable approach; start low (2.5 to 5 cm/H20) and titrate as needed. CPJ\P will direct positive pressure to the lower airways and recruit more alveoli. Invasive positive-pressure ventilation (ie, bag-mask device), if needed, should be performed carefully. Give just enough vol u me to augment the patient's tidal volume, and do not hyperventilate. Address the patient's pain with ketamine IM; narcotics and other CNS depressants can suppre s the patient's respiratory drive. Efforts to stabilize the flail segment with sandbags, IV bags, or bulky dressings are contraindicated as they may further impair chest wall motion, and thus, impair ventilation.

Baroreceptors in the aortic arch and carotid sinuses respond to which of the following? A) Increases in arterial carbon dioxide B) Changes in arterial perfusion pressure C) Increases or decreases in the heart rate D) Fluctuations in the level of arterial oxygen

B) Changes in arterial perfusion pressure Feedback: Baroreceptors, also known as "pressure" receptors, are located within the carotid arteries and aorta. They are extremely sensitive to changes in arterial perfusion pressure (ie, blood pressure). When the baroreceptors sense a drop in arterial blood pressure, they send signals via the sympathetic nervous system, resulting in the release of catecholamines (epinephrine and no,·epinephrine). Catecholamines cause vasoconstriction, resulting in an increase in systemic vascular resistance, as well as increases in heart rate (positive chronotropy) and myocardial contraction force (positive inotropy). Central and peripheral chemoreceptors constantly monitor the pressure of gases (oxygen and carbon dioxide) in the blood, as well as the pH of the cerebrospinal fluid, and help regulate ventilation rate and depth.

Which of the following are consistent with a deep partial-thickness burn? A) Pain, erythema, and destruction of the superficial epidermal layer B) Edema, blister formation, and decreased sensation around the burn C) Charred or leathery skin, absence of pain, and destruction of the dermis D) Blister formation, intense pain, and injury that extends through the dermis

B) Edema, blister formation, and decreased sensation around the burn Feedback: Superficial (first-degree) burns are characterized by pain, erythema (redness), and damage that is li mited to the superficial layer of the epidermis. Partial-thickness (second-degree) burns are categorized as being superfi cial partial-thickness and deep partial-thickness burns. In both types of partial-thickness burn, injury extends through the epidermis and into the dern1is and nuid infiltrates i n between the dermis and epidermis, creating edema and blisters. Unlike a superficial partial-thickness burn, however, a deep partial-thickness burn damages the basal layer of the dermis. As a result, sensation in and around the burn is decreased owi ng to damage to the nerve endings i n the basal layer. A full-thickness (third-degree) burn damages the entire epidermis and dermis, including the nerve endings; this explains why full-thickness burns are usually painless. A full-thickness burn is characterized by charred, white, or leathery skin.

A man was stabbed in the left anterior chest. His BP is 84/58 mm Hg, pulse is 118 beats/min and weak, and respirations are 28 breaths/m in and shallow. He is diaphoretic, his jugular veins are engorged, and his breath sounds are equal bilaterally. What additional clinical finding is MOST consistent with this patient's injury? A) Kehr sign B) Electrical alternans C) Subcutaneous emphysema D) Repeat BP of 90/54 mm Hg

B) Electrical alternans Feedback : This patient's clinical presentation is consistent with a pericardia) tamponade. Beck triad-mufned/distant heart tones, narrowing pulse pressure. and jugular venous distention-is a classic combination of clinical findings in patients with pericardia) tamponade, although it is only observed in approximately 30% of patients. Another classic sign of pericardia) tamponade-albeit one that is not always present-is an ECG finding called electrical alternans. As fluid accumulates within die pericardia) sac, the heart begin to oscillate with each beat. A the heart swings back and forth within the pericardium, its electrical axis changes; this results in QRS compl exes that vary in amplitude (size and height). Kehr sign-referred pain to the left shou lder-is often observed in patients with a splenic inj ury. Subcutaneous emphysema occurs when air infiltrates die subcutaneous (fatty) layer of the skin; it may be observed in patients with injuries such as pneumothorax or tracheal rupture. Relative to an initial BP of 84/58 mm Hg, a repeat BP of 90/54 mm Hg shows a slight widening of the pulse pressure.

In which of the following sit uations would external bleeding be the MOST difficult to control? A) Brachia! artery injury and a systolic BP of 60 mm Hg B) Femoral artery injury and a BP of 160/90 mm Hg C) Popliteal artery injury and a BP of 116/68 mm Hg D) Jugular vein injury and a BP of 80/40 mm Hg

B) Femoral artery injury and a BP of 160/90 mm Hg Feedback: The larger the blood vessel and the higher the blood pressure, the more difficult external bleeding would be to control. Of the i njuries and blood pressures listed, bleeding from a lacerated femoral artery in a patient with a hypertensive BP would be the most difficult to control. The femoral artery is very large (nearly the diameter of you r pinky finger) and hypertension increases the force of blood through the artery, thus increasing the severity of the bleeding. As the blood pressure falls, arterial blood loses it driving force, thus making it easier to control. This is unfortunate, however, because hypotension indicates decompensated shock.

Which of the following forms a substance that holds a clot together and helps achieve hemostasis? A) Plasmin B) Fibrinogen C) Plasminogen D) Angiotensin

B) Fibrinogen Feedback : Bleeding tends to stop rather quickly on its own, usually within about JO minutes, in response to internal clotting mechanisms and exposure to air. When vessels are lacerated, blood flows rapidly from the open vessel. The open ends of the vessel then begin to narrow (vasoconstrict), which reduces the amount of bleeding. Platelets aggregate at the site, plugging the hole and sealing the injured portions of the vessel. This process is called hemostasis. Hemostasis proceeds through three steps: (I) vasoconstriction, followed by (2) platelet aggregation, and finally (3) fibrinogen (factor I) weaving into the clot and forming fibrin to hold the clot together, stopping the bleeding. Bleeding will not stop if a clot does not form. Plasminogen is a pro-enzyme that forms plasmin (also called fibrinolysin) as part of the pathway that breaks down fib1in blood clots (fibrinolysis). Angiotensin is a hormone that causes vasoconstriction and helps regulate blood pressure. It also stimulates the release of aldosterone from the adrenal cortex to promote sodium retention by the kidneys. Angiotensin does not play a role in blood clotting.

A patient is in traumatic shock from an unknown mechanism of i nj ury. Your head-to-toe assessment does not reveal any external evidence of inj ury and his breath sounds are clear to auscultation. Which of the following could explain this? A) Diaphragm rupture B) Hemoperitoneum C) Bowel perforation D) Crushing pelvic injury

B) Hemoperitoneum Feedback : Hemoperitoneum, a collection of blood in the abdominal cavity from either a solid organ or vascular injury, may not immediately be apparent on abdominal examination alone. In fact, this condition often is associated with an initially normal abdominal examination and is identified by the presence of unexplained shock or shock out of proportion to the known injuries. This determination can only be made after a thorough examination that includes all parts of the body, not ju t the abdomen. Bowel perforation i typically not caused by trauma; if it wa , you would expect to find a penetrating inj u ry to the abdomen. A crushed pelvis will be immediately apparent when you palpate the pelvis and find that it is not structu rally intact. Diaphragmatic rupture would be expected to present with diminished breath sounds; on occasion, bowel sounds can be auscultated in the chest if the defect in the diaphragm is large enough. Furthermore, diaphragmatic rupture can present with a scaphoid abdomen (the abdomen has a sunken appearance to it).

Assessment of an inj ured man reveals a closed femur fracture . His vital signs are stable and he is in significant pain. What should you do? A) Apply a traction splint B) Manually stabilize the leg C) Assess distal circulation D) Administer an IV analgesic

B) Manually stabilize the leg Feedback: In a patient with an isolated femur fracture, you would perform all of the interventions listed. However, the most immediate action should be to manually stabilize the leg to prevent further injury. ext, assess distal circulation (ie, pulses, capillary refill). As one can imagine, fractures are painful. Because the patient is hemodynamically stable, you should administer an analgesic to help reduce the inevitable pain that will occur when you have to manipulate the extremity to properly splint it.

In an otherwise healthy 70 - kg adult, hypotension is typically noted after what percentage of blood volume has been lost? A) At least 10% to 15% B) More than 1,500 mL C) Between 15% and 20% D) More than 700 to 800 mL

B) More than 1,500 mL Feedback : The average adult has a total blood volume (TBV) of about 5 li ters. By assessing certain hemodynamic parameters. you can estimate the percentage of blood loss. and thus, the severity of hemorrhagic shock. Hemorrhagic shock is categorized into four classes. A healthy adult can safely tolerate up to 15% (750 mL) TBV loss (Class !). This volume of blood loss falls well within the body's ability to compensate. Loss of 15% to 30% (750 to 1,500 mL) of a person's TBV (Class 1 1 ) requires more sophisticated compensation; it is characterized by tachycardia (>100 beats/min), tachypnea (20 to 30 breath /min), mild anxiety, and a narrowing pulse pressure. The BP, however, i still maintained; thus, the patient is said to be in compensated shock. Hypotension usually appear after the patient has lost more than 30% (1,500 mL) of their TBV (Class Ill). Compensatory mechanisms are failing and the BP can no longer be maintai ned (decompensated shock). The patient has more pronounced tachycardia (>120 beats/min) and tachypnea (30 to 40 breaths/min), narrowed pulse pressure, and marked anxiety or confusion. Survival is possible with early recognition and rapid transport to a trauma center. Class IV hemorrhage occurs when the patient loses more than 40% (2,000 mL) of their TBV. The patient is markedly tachycardic (>140 beats/min), tachypneic (>40 breaths/min), and hypotensive, and is usually lethargic or comatose. Many patients with Cla s IV hemorrhage die i n the prehospital setting because im mediate definitive care is not available. For this reason, they are often said to be in irreversible shock.

You are assessing a trauma patient and suspect that he is experiencing hemorrhagic shock. Which of the following assessment findings would support this suspicion? A) Flushed, moist, cool skin B) Narrowing pulse pressure C) Jugular venous distention D) Slurred or disorganized speech

B) Narrowing pulse pressure Feedback: Signs of hemorrhagic shock include tachycardia, diaphoresis, pallor, restlessness and anxiety, narrowing pulse pressure, and as a later sign, hypotension. Jugular venous distention is observed in patients with a tension pneumothorax or pericardia I tamponade. Collapsed jugular veins, however, are observed in patients with a massive hemothorax, in which case the patient is losing blood in the thoracic cavity. Slurred or disorganized speech is commonly observed in patients with neurologic trauma (ie, head injury) and in patients experiencing a stroke.

An elderly woman slipped on a throw rug and fell, landing on her left hip. She is conscious and alert and in severe pain. Her left knee is flexed and distal circulation is intact. Further assessment reveals no other obvious inj uries and her vital signs are stable. How should you stabilize her injury? A) Carefully straighten her left leg, immobilize her leg with a long rigid splint, and secure her onto a long backboard B) Pad beneath the knee, place her onto a scoop stretcher, pad around her hip, and secure her to the scoop stretcher C) Place her onto a coop stretcher, gently straighten her leg while monitoring her pedal pulse, and bind her legs together D) Apply a pelvic binder device to stabilize her hip , carefully logroll her away from the inj ured side, and place her onto a long backboard

B) Pad beneath the knee, place her onto a scoop stretcher, pad around her hip, and secure her to the scoop stretcher Feedback: Hip fractures and dislocations are common in older adults who have fallen. Remember the first rule of medicine, fi rst do no ha,m? The patient has perfusion distal to her injury. Do not manipulate her leg; doing so will only aggravate her pain and may cause neurovascular compromise. Splint her hip in the position in which it was found. Keep her knee in the nexed position; padding beneath her knee will facilitate this. The scoop (ord1opaedic) stretcher makes an ideal device for moving patients with hip injuries. Unlike the long backboard, it does not require you to logroll the patient and makes padding arou nd the injury easier. /I commercial pelvic binder is used to stabilize an unstable pelvis, not spl int a hip inj u ry.

A man fell 30 feet and landed on a concrete surface. He is responsive to pain only; his BP is 74/50 mm Hg, pulse is 50 beats/min, and respirations are 26 breaths/min. His injuries include a closed deformity to his right femur, numerous abrasions, an open deformity to his right humerus, and deformity in the area of the fifth thorac ic vertebra. Which pathophysiology BEST explains his clinical presentation? A) Internal hemorrhage due to multiple long bone fractures B) Relative hypovolemia due to impaired adrenergic function C) Shock due to failure of the parasympathetic nervous system D) Vascu lar failure secondary to increased alpha-1 stimulation

B) Relative hypovolemia due to impaired adrenergic function Feedback : Neurogenic shock, often the result of a spinal injury, is caused by impaired sympathetic nervous system tone, which results in vasodilation. In neurogenic shock, vascular smooth muscle does not receive impulses that cau e it to contract. As a result, vessel distal to the spinal injury dilate, inc rea ing the size of the vascular space. The normal volume blood can no longer fill the enlarged vascular space. Perfusion of organs and tissues becomes inadequate and hypotension occurs, even though blood loss has not (relative hypovolemia). Bradycardia occurs due to unopposed parasympathetic stimulation and an absence of catecholamine release. Pallor and diaphoresis, signs of sympathetic stimulation and shunting of blood from the periphery, is often observed above the level of the spinal injury. However, the skin below the level of the injury is warm and dry due to vasodilation caused by a loss of vasomotor control. Relative to othe r types of shock, neurogenic shock presents differently; the patient is hypotensive, but does not have the classic signs of pallor, d iaphoresis, and tachycardia.

A man had a strong acid chemical splashed into both of his eyes. He is experiencing intense pain and states that he is wearing contact lenses. What should you do? A) Leave the contact lenses in and beginning irrigation of both eyes B) Remove the contact lenses and beginning irrigation of both eyes C) Leave the contact lenses in and cover both eyes with sterile gauze D) Remove the contact lenses and cover both eyes with sterile gauze

B) Remove the contact lenses and beginning irrigation of both eyes Feedback: In general, contact lenses should be left in place. The exception to this is a chemical eye injury. You m ust remove the contact lenses and then flush the eyes with copious amounts of water. If the contact lenses are left in place, the chemical can become trapped behind them, which could result in continued burning of the eyes.

A man has a large knife impaled in the precordial area, just to the left of the angle of Louis. He is unresponsive, apneic, and pulseless. What should you do? A) Carefully stabilize the knife in place and begin CPR B) Remove the knife, control bleeding, and begin CPR C) Lift the knife j ust enough so that you can begin CPR D) Carefully remove the knife and assess his cardiac rhythm

B) Remove the knife, control bleeding, and begin CPR Feedback: There are two indications for removing impaled objects: when they compromise the airway and/or impede breathing, and when they interfere with your ability to perform CPR. Because chest compressions are performed in the precordial area, you must remove the knife, control any external bleeding, and begin CPR. After CPR has been initiated, you should assess the patient's cardiac rhythm and treat accordingly.

A patient struck the steering wheel when his vehicle crashed into a tree. Assessment reveals respiratory distress, diminished breath sounds to the left lower hemithorax, a scaphoid abdomen, and jugular venous distention. What should you suspect? A) Ma sive hemothorax B) Ruptured diaphragm C) Pericardia! tamponade D) Tension pneumothorax

B) Ruptured diaphragm Feedback : Injuries to the diaphragm, although rare, result from both blunt and penetrating trauma. Blunt force trauma following a high-speed motor vehicle crash is one of the most common mechanisms of injury. During a frontal impact, the unrestrained patient strikes the steering wheel or column, causing a sudden increase in intraabdominal pressure that may tear the diaphragm. Because the left side of the diaphragm is embryonically weak and the right side is protected by the liver, rupture - particularly those caused by blunt trauma-are more common on the left side. In most cases, a portion of the large intestine enters the thoracic cavity through the diaphragmatic tear, causing ventilatory impairment by compressing the lung. In larger diaphragmatic tears, cardiac output may be decreased secondary to impaired ventricular filling, resulting in hypotension, tachycardia, and increased jugular venous pressure. Signs of diaphragmatic rupture include labored breathing, asymmetric chest wall movement, jugular venous distention, and a scaphoid abdomen (the abdominal wall is sunken and presents with a concave rad1er than a convex contour). In some cases, bowel sou nds can be heard in the lower to middle part of one of the hemithoraces: however, unilaterally diminished breath sounds are more common.

A 120-pound woman has partial- and full-thickness burns to her head, face, and anterior chest. She is tachypneic, tachycardic, and hypotensive. When caring for her, you should: A) recognize that 27% of her body surface area has been burned. B) suspect that she has internal hemorrhage from an occult injury. C) cover her burns with moist, sterile dressings to relieve any pain. D) give a 250 -mL normal saline bolus during transport to the hospital.

B) suspect that she has internal hemorrhage from an occult injury. Feedback : Unless accompanied by other injuries (eg, occult hemorrhage, multiple long bone fractures), burn patients are uncommonly hypovolemic in the prehospital setting; hypovolemia typically occurs over the first several hours following the burn (burn shock). If the burn patient has signs of shock in the field, you should suspect internal hemorrhage from an occult injury. This patient has burns that cover 18% of her body surface area (anterior chest: 9%, head and face: 9%). Because of the risks of infection and hypothermia, partial- and full-thickness b u rns should be covered with dry, sterile dressings. If the patient is in shock, you should give sufficiem amounts of an isotonic crystalloid solution- usually 20 mL/kg-to maintain adequate perfusion (1,100 mL in a 120-pound [55-kg] patient).

During your assessment of an unresponsive 21-year-old male with a suspected spinal inj ury, you test for a Babinski reflex. A Babinski reflex is present if: A) the toes curl downward when a blunt object is stroked along the sole of the foot. B) the big toe turns upward when a blunt object is stroked along the sole of the foot. C) a blunt stimulus applied to the sole of the foot causes dorsiflexion of the entire foot. D) the big toe curls downward when a blunt object is stroked along the sole of the foot.

B) the big toe turns upward when a blunt object is stroked along the sole of the foot. Feedback: Any unresponsive trauma patient - especially one with a suspected head or spinal injury - should be assessed for his or her response to pain, for decerebrate and decorticate posturing (indicates brain stem injury), and for the p resence of a Babinski reflex, also referred to as the plantar response. A Babinski reflex is present if the big toe turns upward when a blunt object is stroked along the sole of the foot; it is absent if the big toe turns downward. The Babinski reflex is not reported as being positive or negative; it is either present or absent. In newborns and small infants, a present Babinski reflex exists as a primit ive reflex and is therefore a norrnal finding. In older children and adults, however, it is grossly abnormal and indicates central nervous system injury.

A man opens his eyes when you speak to him and pulls his arm away when it is palpated. He knows his name, but cannot remember what happened and does not know what day it is. What Glasgow Coma Scale score should you assign? A) 9 B) 1O C) 11 D) 12

C) 11 Feedback: The patient's GCS should be documented as II. He opens his eyes in response to verbal stimuli (eye opening, 3), he is confused (verbal response, 4), and he withdraws from pain (motor response, 4).

Which of the following vehicle impacts will create the greatest amount of kinetic energy? A) 160-lb patient who is traveling at 45 mph B) 170-lb patient who is traveling at 55 mph C) 150-lb patient who is traveling at 60 mph D) 180 -lb patient who is traveling at 50 mph

C) 150-lb patient who is traveling at 60 mph Feedback: Kinetic energy (KE) is a function of an object's mass (weight) and velocity (speed). The greater than amount of KE, the greater the potential for serious injury or death. The relationship between weight and speed as it affects KE is as follows: KE equals one-half the mass times the velocity square (KE = t;2mv2). According to this formula, doubling the mass doubles the amount of KE; however, doubling the velocity quadruples the amount of KE. A 150-lb patient who strikes a fixed object while traveling at 60 mph will create 270,000 units of KE, as follows: 150 + 2 (75) , 60 squared (3,600) = 270,000. A 160-lb patient traveling at 45 mph will create 162,000 units of KE, a 170-lb patient traveling at 55 mph will create 257,125 units of KE, and a 180-lb patient traveling at 50 mph will create 225,000 units of KE.

Which of the following parameters yields the lowest cerebral perfusion pressure? A) BP, 96/50 mm Hg, intracranial pressure, 5 mm Hg B) BP, 100/54 mm Hg; intracranial pressure, 10 mm Hg C) BP, 105/60 mm Hg; intracranial pressure, 20 mm Hg D) BP, 130/90 mm Hg; intracranial pressure, 25 mm Hg

C) BP, 105/60 mm Hg; intracranial pressure, 20 mm Hg Feedback : Cerebral perfusion pressure (CPP) is the amount of pressure that it takes to push blood through the cerebral ci rculation and maintain blood flow and oxygen and glucose delivery to the cells of the brain. If the CPP drops below 60 mm Hg, cerebral perfusion will be compromised. Although the paramedic cannot measure CPP in the field, he or she shou ld know how to calculate and interpret it. This knowledge will facilitate an appreciation of the disastrous effects that hypotension and/or increased intracranial pressure (ICP) can have on CPP. CPP decreases when the mean arterial pressure (MAP) dec reases and/or the ICP increases (ICP should not exceed 15 mm Hg). Even a single episode of hypotension in the patient with a traumatic brain injury can be catastrophic. CPP is calculated as follows: CPP = MAP - ICP. MAP is calculated as follows: MAP = systolic BP + (diastolic BP • 2) + 3. A BP of 105/60 mm Hg ( MAP of 75) with an ICP of 20 mm Hg yields a CPP of 55 mm Hg, which is the lowest of the options listed.

Which of the following sets of vital signs is consistent with hemorrhagic shock in an otherwise healthy patient? A) BP, 80/40 mm Hg; pulse, 70 beats/min; respirations, 24 breaths/min B) BP, 190/100 mm Hg; pulse, 50 beats/min; respirations, 8 breaths/min C) BP, 90/50 mm Hg; pulse, 120 beats/min ; respirations, 28 breaths/min D) BP, 160/70 mm Hg; pulse, 140 beats/min; respirations, 12 breaths/min

C) BP, 90/50 mm Hg; pulse, 120 beats/min ; respirations, 28 breaths/min Feedback: A patient with hemorrhagic shock would be expected to be hypotensive, tachycardic, and tachypneic. Hypotension and normocardia (or bradycardia) are more consistent with neurogenic shock. Of course, patients who take beta blockers may not present with tachyca rdia (or may only have a mild increase in heart rate) in response to shock. Hypertension, bradycardia, and bradypnea are consistent with increased intracranial pressure. Hypertension and tachycardia could indicate a hypertensive emergency or may be a response of the body to fear or pai n.

Why do subdural hematomas in older adults have a higher mortality rate than in younger adults? A) Alterations in mental status caused by a head injury are typically attributed to dementia. B) Older adults have a lower relative blood volume and are more susceptible to hypovolemia. C) Because of brain atrophy, a large subdural hemorrhage can exist with minimal clinical signs. D) The subdural space decreases with age, so intracranial pressure increases more rapidly.

C) Because of brain atrophy, a large subdural hemorrhage can exist with minimal clinical signs. Feedback: Because of age-related brain atrophy, a large subdural hemorrhage can exist with minimal clinical findings. By the time clinical signs manifest, the injury may have reached catastrophic proportions. Younger adu lts would likely develop symptoms much earlier; even then, subdural hematomas can take up to 24 hours to cause symptoms. All contents in the skull get smaller with age; this means that the cranium can hold a larger volume of blood. Therefore, in addition to intracranial pressure, it is possible for the older adult with a large subdural or epidural hematoma to develop hypovolemic shock as well. This combination of head trauma and hypovolemic shock yields a higher mortality rate. Blood volume remain proportionate to a person's body size; ol der adult do not have a lower relative blood volume. Any alteration in mentation following a head injury in an older adult should NEVER be attributed to anything other than a traumatic intracranial pathology

A patie nt has extensive full-thickness burns. H is BP is 1 54/88, pulse is 112 beats/min, and respirations are 20 breaths/min. What should you do? A) Cool the burns with steri le saline or water B) Apply a cold, wet blanket to the patient C) Continuously assess the patient's airway D) Administer sequential IV fluid boluses.

C) Continuously assess the patient's airway Feedback: A common cause of death in patients with extensive burns, especially full-thickness burns. is swelling and closure of the ai rway. You must constantly monitor the patient for signs of swelling/closure, which include singed nasal hairs, a brassy cough, hoarseness, stridor, and progressive respiratory distress. In addition to monitoring the airway, the patient must be protected from hypothermia. Do not attempt to cool full-thickness burns; doing so increases the risks of hypothermia and infection. Full-thickness burns should be covered with dry, sterile dressings or a sterile burn heet. Unless associated with occult internal bleeding from another injury, burn patients are typically not hypotensive in the field; therefore, IV fluid boluses are usually not i ndicated. Burn shock, which occurs when fluid shifts from the vasculature into the soft tissues, often does not occur unti l 8 to 12 hours after the patient was burned.

Treat ment for a full-thickness burns includes which of the following? A) Covering the burns with moist, sterile dressings and administering analgesia B) Irrigating the burns with a sterile saline solution and establishing IV access C) Covering the bums with a dry, sterile dressing and keeping the patient warm D) Applying an antibiotic ointment or cream to the burns and rapidly transporting

C) Covering the bums with a dry, sterile dressing and keeping the patient warm Feedback : Because full-lhickness (third-degree) burns destroy all layers of the skin, the patient is prone to hypothermia and infection. After tending to any airway, breathing, and circulation issues, you should cover the burns and keep the patient warm. Use dry sterile dressings or a sterile burn sheet to cover the burns, and then cover the patient with a blanket. Do not irrigate a full-thickness burn or apply moist dressings; these actions increase the risks of hypothermia and infection. Burn ointments and creams should also be avoided; these will only have to be removed at the hospital and may increa e the risk of infection. Transport the critically burned patient prom ptly and establish vascular acce s en route. Administer oxygen as needed and carefully monitor the status of the patient's airway. Fu ll-thickness burns alone are usually pain less owing to nerve destruction; however, partial-thickness burns are often present arou nd the full-thickness burn and can be intensely painful. If the patient's hemodynamic status permits, administer analgesia.

What impact does hypothermia have on a patient with severe traumatic hemorrhage? A) Increased heart rate B) Decreased tissue acidosis C) Decreased blood clotting D) Increased metabolic rate

C) Decreased blood clotting Feedback : Hypothermia, one of the three components of the trauma triad of death, contributes to coagulopathy (impaired blood clotting), which reduces survival from traumatic hemorrhage. If the blood cannot clot, the body cannot slow or stop the bleeding on its own (hemostasis). Therefore, any factor that interferes with blood clotting will cause the patient to lose more blood than would occur with adequate clotting. This will lead to poor perfusion and, ultimately, death. Acidosis (blood pH of less than 7.35) often occurs with both excessive hemorrhage and some treatments perforrned to compensate for it. For example, normal saline i acidic and can increase acidosis; thi is one reason why crystalloid fluid boluses hould be avoided in patients with severe hemorrhage. Acidosis contributes to coagulopathy and complicates treatment of the patient. Hypothermia decreases, not increases, the metabolic rate. As metabolism decreases, the heart rate would be expected to decrease as well. Keep trauma patients warm!

By which mechanism does a tension pneumothorax cause circulatory compromise? A) Respiratory failure B) Massive hemorrhage C) Decreased preload D) Increased afterload

C) Decreased preload Feedback: Tension pneu mothorax can be looked at as a "refill" problem. As pleural tension increases, compression of the venae cavae cause a decrease in venous return, resulting in inadequate refill of the right atrium. Venous return contributes directly to preload, or end diastolic pressure, which is the amount of pressure inside a cardiac chamber right before it contracts. While it is true that many patients with a tension pneumothorax experience varying degrees of respiratory distress or insufficiency, this is not the pr imary mechanism that causes circulatory collapse and shock. Most tension pneumothoraces are not associated with massive intrathoracic hemorrhage. Afterload, the resistance against which the heart must contract, increases when there is resistance to the forward flow of blood (ie, hypertension). Tension pneumothorax does not increase afterload, but rather reduces preload.

A man with blunt abdominal trauma has a BP of 60/40 mm Hg, a heart rate of 124 beats/min, and a respiratory rate of 22 breaths/min. What is the appropriate crystalloid fluid resuscitation for him? A) 500- to 750-mL boluses every 5 to 10 minutes B) Enough to maintain a systolic BP of 100 mm Hg C) Enough to maintain a MAP of 60 to 65 mm Hg D) Crystalloids should not be given to this patient.

C) Enough to maintain a MAP of 60 to 65 mm Hg Feedback : The best replacement fluid for lost blood is, of course, blood. However, this is not available in all EMS systems, although it is becoming increasingly popular. For patients with suspected internal hemorrhage in the chest, abdomen, or pelvis, IV crystalloid solution should be titrated to maintain a systolic BP of 80 to 90 mm Hg, which will provide a mean arterial pressure (MAP) of 60 to 65 mm Hg. A MAP of 60 to 65 mm Hg should maintain adequate perfusion to the vital organs with less risk of worsening internal hemorrhage. A fluid bolus should not be administered because this may increase the BP too m uch a nd exacerbate the hemorrhage. The cu rrent philosophy of restricted crystalloid ad ministration in the prehospital sett ing has been called by several na mes, including permissive hypotension, hypotensive resuscitation, and "balanced" resuscitation, meaning that a balance must be struck between the amount of fluid administered and the degree of blood pressure elevation. The days of barraging trauma patients with crystalloid fluids, especially those with uncontrolled internal hemorrhage, have long since passed!

Which of the following inj uries would likely result from a motorcycle striking a fixed object? A) Chest and abdominal trauma B) Tibia, fi bula, and pelvic fractures C) Femur fractures and head injury D) Thoracic spine and femur fractures

C) Femur fractures and head injury Feedback: If you can imagine a motorcycle striking a fixed object, the operator will be ejected, striking their legs on the handlebars, which fractures one or both of the femurs. The operator then typically strikes the ground or object head first. Motorcycle helmets arguably decrease the risk of a serious head injury; however, they do not prevent a head injury altogether.

A man has a large open wound to his upper thigh that is bleeding profusely. What should you do? A) Administer high-flow oxygen and cover him with a warm blanket B) Cover the wound with a dressing and apply a tourniquet proximal to the wound C) Find his femoral artery with your finger and compress it against a solid structure D) Apply ice to the wound and elevate his leg to try to slow the bleeding

C) Find his femoral artery with your finger and compress it against a solid structure Feedback: Massive hemorrhage control begins with your finger! If the wound is large, locate the artery proximal to the wound and push it against a solid structure, such as the femur. After gaining digital control of the bleeding, apply a tourniquet proxi mal to the wound. After all massive hemorrhage has been controlled, treat the patient for shock (ie, oxygen, thermal management). The goal of massive hemorrhage control is to STOP the bleeding, not merely slow it down. A patient who is bleeding slowly is still bleeding!

According to current G u idelines for Field Triage of Inj ured Patients, which of the following patients should be transported to a trauma center that provides the highest level of care within the defined trauma system? A) Side impact with 6 inches of vehicle intrusion; GCS of 15; heart rate of 80 beats/min B) Scalp laceration without skull deformity; GCS of 14; systolic BP of 130 mm Hg C) Fractured humerus and femur; systolic BP of 110 mm Hg; heart rate of 120 beats/min D) Bruising to the mid sternum; systolic BP of l20 mm Hg; heart rate of llO beats/min

C) Fractured humerus and femur; systolic BP of 110 mm Hg; heart rate of 120 beats/min Feedback : According to the 2011 Guidelines for Field Triage of Injured Patients, published by the Centers for Disease Control and Prevention (CDC), the paramedic should assess four components when determining the appropriate transport destination for a trauma patient: physiologic criteria, anatomic criteria, mechanism of injury, and special patientjsystem considerations. According to the guidelines, two or more proximal long bone fractures meet the anatomic criteria for preferential transport to a trauma center that provides the highest level of care within the defined trauma system. Other anatomic criteria include open or depressed skull fractures, pelvic fractures, and chest wall instability or deformity (ie, flail chest). Physiologic criteria include a systolic BP le s than 90 mm Hg or a Glasgow Coma Scale (GCS) score that is less than or equal to 13, among others. A high-risk vehicle crash includes intrusion (including the roof) of greater than 12 inches to the occupant site or greater t han 18 inches to any site. Refer to the 2011 CDC Guidelines for Field Triage of Injured Patients for more information.

A woman was rescued from her burning house. She is confused and complains of a severe headache and dizziness. Her heart rate is 130 beats/min and her respiratory rate is 36 breaths/min . Which of the following is indicated for her? A) Atropine B) Pralidoxime C) Hydroxocobalamin D) Sodium bicarbonate

C) Hydroxocobalamin Feedback : Patients removed from a structure fire should be treated for carbon monoxide (CO) but should also be considered at risk for cyanide toxicity. Cyanide gas is produced from t he burning of plastics or polyurethane. Cyanide poisons the process of energy production, preventing the body from using oxygen. The patient with cyanide toxicity can die from asphyxia despite having adequate amounts of o>.-ygen available in the blood. Sign and symptoms of cyanide toxicity include altered level of consciou nes , dizziness, headache, tachycardia, and tachypnea. This patient should clearly receive high-flow oxygen; however, you should also administer an antidote for cyanide. The preferred antidote for cyanide toxicity is a medication t hat directly binds to the cyanide molecule, rendering is harmless. Hydroxocobalamin (Cyanokit) detoxifies the cyanide by directly binding to it and forming cyanocobalamin (vitamin B12), which is nontoxic. Atropine is indicated for symptomatic bradycardia and organophosphate poisoning; tJ1is patient has neither. Pralidoxime (2-PAM) is used for chemical nerve agent exposure. Sodium bicarbonate is not used for CO or cyanide toxicity; it is u sed to treat metabolic acidosis.

You are assessing a man who was the unrestrained driver of a small car that struck a bridge pillar while traveling at a high rate of speed. He is diaphoretic, he has unlabored tachypnea, and his pulse is rapid and thready. Lung sounds are clear and equal bilaterally. You do not see any visible trauma to the patient's head or face. What should you suspect? A) Increased intracranial pressure B) Severe diaphragmatic rupture C) Intraabdominal hemorrhage D) Tension hemopneumothorax

C) Intraabdominal hemorrhage Feedback : /Is evidenced by the cool, clammy skin; tachypnea; and rapid, thready pulse, it is clear that this patient is in shock-most likely from intraabdominal hemorrhage due to abdominal impact wid1 the steering wheel. The presence of clear and equal breath sounds bilaterally, plus the fact that his respirations-albeit tachypneic-are not labored, rules out a tension pneumothorax, mas ive hemothorax, or hemopneumothorax. Severe diaphragmatic rupture is an unlikely cause; patients with this type of injury also commonly present with l abored breathing. Furthermore, patients with a ruptured diaphragm may present with bowel sounds d1at are heard when auscultating the lungs, depending on dle size of the diaphragmatic rupture. The patient's signs and symptoms are not consistent with increased intracranial pressure (ICP) due to a traumatic brain injury (TBl); such patients typically present with hypertension, bradycardia, and irregular respirations.

A trauma pat ient has signs of shock, but no external signs of inj ury. What should you suspect? A) Bleeding into the pelvic cavity B) Intrathoracic hemorrhage C) Intraabdominal hemorrhage D) Intracerebral hemorrhage

C) Intraabdominal hemorrhage Feedback: Because bleeding into the retroperitoneal space may not produce the obvious signs of abdominal injury (ie, distention, rigidity, bruising, etc), trauma patients with unexplained shock should be assumed to have an intraabdominal hemorrhage. The retroperitoneal space is a common area for hidden bleeding. lntracerebral hemorrhage would be expected to present with evidence of increased intracranial pressure, not signs of shock. Hemorrhage into the pelvis or thoracic cavity typically presents with external evidence of trauma, such as abrasions, bruising, or crepitus.

Why are older women more likely to s ustain a serious inj ury from a fall than older men? A) Slower reflexes B) Lower blood volume C) More pronounced osteoporosis D) Age-related atrophy of the brain

C) More pronounced osteoporosis Feedback: Falls are the leading cause of traumatic death and disability in older adults. Although men and women fall with equal frequency, women are more than twice as likely to sustain a serious injury because of more pronounced osteoporosis, which makes their bones even more prone to fracture. Men and women have the same relative blood volume. There is no known difference between men and women regarding how fast the brain atrophies with age. There is also no known difference in reflex time between men and women.

An unrestrained woman struck the steering wheel when her car hit a tree. She reports pain to the midsternal area, which is point tender to palpation. Her BP is 100/60 mm Hg, pulse is 118 beats/min and irregular, and respirations are 26 breaths/min and shallow. What should you suspect? A) Flail chest B) Pericardia! tamponade C) Myocardial contusion D) Esophageal inj ury

C) Myocardial contusion Feedback: The mechanism of and location of the injury, as well as the irregularity of the patient's pulse, are suggestive of a myocard ial contusion. Patients with this type of injury can experience all of the same deleterious effects associated with an acute myocardial infarction, including cardiogenic shock, arrhythmias, and cardiac arrest. Although a flail chest, pericardia) tamponade, or esophageal injury cannot be completely ruled out in the field, there are no physical exam findings that suggest any of these injuries.

Which type of fracture commonly occurs following direct or twisting forces? A) Linear B) Spiral C) Oblique D) Transverse

C) Oblique Feedback: An oblique fracture is one in which the fracture occurs at an angle across the bone; direct or twisting forces commonly cause this type of fracture. Conversely, spiral fractures, fractures that incircle the bone, are exclusively caused by twisting forces. Linear fractures occur parallel to the long axis of the bone; low-energy stress inju ries commonly cause linear fractures. Transverse fractures occur straight across a bone at right angles to the cortex; they are commonly the result of a direct, low-energy force. Not all fractures cause deformity (linear fractures, for example); therefore, you cannot rule out a fracture based on an absence of deformity.

A 6-inch metal rod perforated a man's cheek and is impaled in the hard palate of his mouth. The patient is conscious and alert and is breathing without difficulty. There is a small amount of bleeding into his mouth. What should you do? A) Gently twist the rod out of the hard palate and remove it in the same direction it entered B) Cut the part of the rod that is protruding out of his mouth and then po ition him on his side C) Stabilize the rod in place with bulky dressings and allow the patient to suction his own mouth D) Gently twist the rod out of the hard palate and remove it in the opposite direction it entered

C) Stabilize the rod in place with bulky dressings and allow the patient to suction his own mouth Feedback: An impaled object should be removed if it impairs a patient's airway or ability to breathe. However, in this case, the patient is conscious and alert, is breathing without difficulty, and is maintaining his own airway. Removing an impaled object, although sometimes necessary, is risky. In this case, it would be in the patient's best interest to stabilize the metal rod in place and allow the patient to self-suction. Cutting the rod would likely cau e further injury to the hard palate and potentially worsen the bleeding. Carefully monitor the patient's airway status and transport without delay.

A man was near a building when it exploded. He presents with widespread burns, hemoptysis, and blunt head trauma. During which phase of the explosion did he likely expereince his head injury? A) Primary B) Secondary C) Tertiary D) Quadernary

C) Tertiary Feedback: Injuries during an explosion can occur by any of four mechanisms. Primary blast injuries, those that are caused by the pressure wave of the explosion itself, include rupture of hollow organs and the tympanic membranes. Secondary blast injuries are caused by debris or fragments from the explosion; such injuries include lacerations and impaled objects. The tertiary blast injury is produced when the victim is propelled away from the explosion and strikes another object; it is during this phase chat blunt injuries to the head, spine, and chesl typically occur. Quadernary blast injuries include burns and respiratory injuries from hol gases or chemicals.

A man experienced blunt abdominal trauma and presents with tachycardia, weak radial pulses, diaphoresis, and restlessness. His BP is 74/60 mm Hg. You should administer crystalloid fluids until: A) a decrease in heart rate is noted. B) his skin becomes warm and dry. C) his radial pulses are easily palpable. D) his systolic BP is at least 100 mm Hg.

C) his radial pulses are easily palpable. Feedback : IV fluid therapy can be helpful in supplementing shock treatment (eg, oxygen, thermal management) in patients with controlled external bleeding. However, IV fluids should be administered with caution in patients with hypovolemic shock caused by bleeding that cannot be controlled (eg, internal bleeding). The patient in this scenario has obvious signs of shock, which is likely the result of intraabdominal hemoJThage. Internal hemorrhage cannot be controlled in the prehospital setting; it requires prompt surgical intervention. Therefore, the goal of volume replacement is to maintain perfusion without increasing inte rnal bleeding. A rapid increase in blood pressure caused by excessive volume replacement can interfere with hemo tasis-the body's natu ral physiologic process that slows or halts bleeding. For this reason, most protocols advise administration of just enough isotonic crystalloid fluids (eg, normal saline, lactated Ringer) to improve perfusion (ie, stronger radial pulses, improved mental status). Studies suggest that fluid therapy to maintain the systolic BP at approximately 80 mm Hg-provided that the patient's mental status is adequateis safer than attempting to restore normotension, which may aggravate ongoing bleeding. Remember, the goal of volume replacement is to maintain adequate perfusion, not increase blood pressure. Of course, whole blood would be ideal in the setting of internal hemorrhage.

Immediate care for a severely burned patient includes: A) stopping the burning process. B) applying dressings to the burns. C) moving the patient to safety. D) ensuring a patent airway.

C) moving the patient to safety. Feedback: Prior to providing care to ANY patient, you must ensure that you and the patient are in a place of afety. Once this is ensured, begin the assessment and treatment process. In Lhe burn patient, your next priority is to stop the burning process.

Which of the following inj ury mechanisms involves axial loading? A) A driver's head is thrust backward after his vehicle is struck from behind by another vehicle B) A lateral impact motor vehicle crash in which the driver's head is pulled to the side as her torso moves C) A fall from a ladder in which the patient strikes his chin on one of the rungs before striking the gro und D) A rollover motor vehicle crash in which the unrestrained occupant strikes his head on the interior of the roof

D) A rollover motor vehicle crash in which the unrestrained occupant strikes his head on the interior of the roof Feedback: Axial loading causes vertical compre sion of the spine when direct forces a re transmitted down the spinal column. The point of impact may be the head or the feet. Significant mechanisms of injury involving axial loading often cause compression fractures or crushing of one or more of the vertebral bodies; spinal cord injury may or may not occur. Common mechanisms of injury that cause axial loading include striking the top of the head on the interior roof of a vehicle during a rollover motor vehicle crash, falls from a significant height in which the patient lands on their feet, diving head-first into shallow water, and a direct blow to the top of the head with a heavy object. one of the other injury mechanisms described involve vertical compression of the spine.

A middle-aged male experienced partial-thickness splash burns to 36% of his body surface area. The burns are all located above his waist. What parts of his body have bee n burned? A) Abdomen and one arm B) Chest, head, and one arm C) Chest, neck, and both arms D) Anterior torso and both arms

D) Anterior torso and both arms Feedback : According to the adult Rule of Nines, the head (including the face and neck) represents 9% of the total body surface area (TBSA), the anterior torso (chest and abdomen) represents 18% of the TBSA, and each entire upper extremity represents 9% of the TBSA. Given that all of the patient's bums are located above the waist, the only combination in this question that equals 36% is the anterior torso (18%) and both arms ( 18%). The chest is one-half of the torso; therefore, it represents 9% of the TBSA. The abdomen is also one-half of the torso; therefore, it also represents 9% of the TBSA. Burns to the abdomen and one arm would equal 18% of the TBSA. Burns to the chest, head, and one arm would equal 27% of the TBSA. Burns to the chest, neck, and both arms would equal ± 30% of the TBSA.

Which of the following spinal cord injuries presents with weakness of the upper and lower extremities on the ipsilateral side and loss of pain and temperature sensation on the contralateral side? A) Central cord syndrome B) Anterior cord syndrome C) Complete cord transection D) Brown-Sequard syndrome

D) Brown-Sequard syndrome Feedback : Brown-Sequard syndrome is caused by partial transection (hemitransection) of the spinal cord, typically from intervertebral disk rupture or an unstable vertebral fragment that puts pressure on the spinal cord. Classically, pressure on half of the spinal cord causes extremity weakness on the ipsilateral (same) side as the cord injury, and loss of pain and temperature sensation on the contralateral (opposite) side. In central cord syndrome, the patient experiences greater motor deficit to the upper extremities; he or she often has paralyzed arms, but retains the ability to move their legs. Anterior cord syndrome is caused by pressure on die anterior part of the spinal cord by a vertebral fragment that is forced posteriorly into the spinal canal; signs and symptoms include decreased sensation of temperature and pain distal to the injury, while light touch and position sensation (proprioception) remain intact. Complete cord transection results in loss of all sensory and motor function distal to the site of the injury; nothing is spared.

man fell 20 feet and landed on his feet. You should suspect inj uries to which of the following? A) Knees, pelvis, and thoracic spine B) Tibia/fibula, pelvis, and cervical spine C) Heels of the feet and cervical spine D) Calcaneus, hips, and lumbar spine

D) Calcaneus, hips, and lumbar spine Feedback: A typical pattern of injury is often observed when an adult falls and lands on their feet. As a result of axial loading, the patient frequently sustains inj ury to the calcaneus (heels of the feet) as the initial point of impact. The force then travels upwa rd, causing injury to the hips. The force can be severe enough to result in injury to the relatively unprotected lumbar spine.

A man has a large open injury to his inner thigh that is bleeding heavily. His skin is pale and he is screaming in pain. What should you do? A) Administer 2 mg/kg of ketamine IM B) Apply a tourniquet 2 to 4 inches above the wound C) Immediately cover the inj ury with a multitrauma dressing D) Compress the bleeding vessels against his femur with your fingers

D) Compress the bleeding vessels against his femur with your fingers Feedback: Control of massive hemorrhage from an open wound begins with your finger! Expose die wound and compress the bleeding vessels against a bony strncture (in this case, the femur). Sweep the blood from the wound after doing so to ensure that you have stopped the bleeding. /\ proximal tourniquet should then be applied ; the more proximal the better. Simply covering the wound with dressings will do little more than hide the bleeding and is a waste of the patient's time. After the bleeding has been stopped, you can address the patient' s pain if he or she is hemodynamically stable. Ketam ine is a useful drug because it does not lower the blood pressure.

Treatment for suspected compartment syndrome in a patient with a lower extremity fracture includes which of the following? A) 1 gram of calcium chloride B) I mEq/kg of sodium bicarbonate C) Elevating the extremity 12 inches D) Dorsiflexion of the foot when splinted

D) Dorsiflexion of the foot when splinted Feedback: In the hospital, com partment pressures can be measured in extremities where compartment syndrome is suspected, and a fasciotomy is the definitive management for this condition. Only basic maneuvers can be attempted in the field. Any tightly applied splint or dressings should be removed, and distal perfusion reassessed. Splinting the extremity provides stability. Elevation of the extremity is not recommended. Keeping the extremity level with the heart is ideal . Additionally, the foot should be dorsiflexed when splinted to reduce anterior compartment pressure on the lower leg. either calcium nor sodium bicarbonate is indicated for compartment syndrome; however, they may be given to patients with crush injuries before the compressive force is removed. Because compartment syndrome can develop duri ng a long-distance tra nsport, serial exami nations are essential for early identi fication of this problem.

A hiker ascended 13,000 feet up a mountain in less than a day. The next morning, he is confused and is unsteady on his feet. What should you suspect? A) Hypoglycemia B) Ac ute ischemic stroke C) Subarachnoid hemorrhage D) High-altitude cerebral edema

D) High-altitude cerebral edema Feedback : H igh-altitude cerebral edema ( HACE) is a very serious neurologic syndrome that can develop in individual s who ascend to certain heights. At altitudes above 8,000 feet (2,440 m), cerebral blood flow increases because of hypoxia-induced vasodilation. The mechanism of injury appears to be related to a combination of sustained cerebral vasodilation, increased capil lary permeability across the blood-brain barrier, and the inability to compensate for the excess cerebral edema. HACE can occur at a ny time within 3 to 5 days after arrival at 9,000 feet, but generally it occurs at altitudes above 12,000 feet, with an onset of symptoms within hours. Hallmark feature of HACE are altered level of consciousness and ataxia (staggering gait}, along wid1 drowsiness, stupor, and confusion progressing to coma. Death resul ts from brain herniation. Any of the other conditions l isted are possible; however, given the fact that he just ascended to a great height in a short period of time, HACE is the most likely etiology of his presentation.

A construction worker was exposed to a strong alkali chemical. Tissue damage may occur through which of the following mechanisms? A) Hyperkalemia B) Myoglobinemia C) Coagulation necrosis D) Liquefaction necrosis

D) Liquefaction necrosis Feedback : Acids are agents with a pH between 7 (neutral) and O (strong acid). Alkalis (bases) are agents with a pH between 7 and 14 (strong base). Alkali burns are generally worse than acid burns. Acids damage tissue by a process called coagulation necrosis; the damaged tissue coagulates and transforms into a barrier that prevents deeper penetration of the acid. In contrast, alkali burns destroy the tissue by liquefaction necrosis; the base liquifies the tissue, allowing the chemical to penetrate more deeply and cause increasingly deeper tissue damage. Alkali agents dissolve proteins of tissue and form alkaline proteins, which are soluble and allow further reaction deeper into affected tissues. Crush injuries and electrical burns can cause an increase in serum potassium (hyperkalemia). Hyperkalemia does not damage the tissues; it increases the risk for life-threatening dysrhythmias. Likewise, myoglobin can be released into the bloodstream following crush injuries or electrical burns. Myoglobin can obstruct the renal tubules and cause acute renal failure.

Which of the following inj uries would likely cause obstructive shock? A) Pelvic fracture B) Crushed trachea C) Simple pneumothorax D) Pericardia! tamponade

D) Pericardia! tamponade Feedback: Obstructive shock results from an injury or condition that obstructs the flow of blood and causes inadequate circulation through the card iovascular system. Injuries that can result in this type of shock include pericardia! tamponade, tension pneumothorax, and pulmonary embolism. In a pericardia! tamponade, blood within the pericardia I sac inhibits cardiac relaxation, which causes a reduction in venous return. Contractility is also decreased, resulting in decreased cardiac output. In a tension pneumothorax, excessive pleural pressure squeezes the heart and compresses the vena cavae; this causes decreases in venous return and cardiac output. In a pulmonary embolism, a clot in a pulmonary artery obstructs the flow of blood to the lungs, which inhibits pulmonary (external) respiration.

By which of the following mechanisms does a pericardia! tamponade cause hemodynamic compromise? A) Massive hemorrhage B) Aortic compression C) Vena cava compression D) Reduced venous return

D) Reduced venous return Feedback: When blood accumulates in the pericardia! sac, it restricts the relaxation phase of the heart (diastole). If the heart cannot relax, it cannot refill with blood. As a result, venous return is impaired (a "refill" problem, so to speak). If the right side of the heart does not fill, there is nothing to send through the lungs to the left side of the heart; as a result, cardiac output decreases. Patients with an isolated pericardia] tamponade do not bleed to death; they simply cannot refill their heart with the blood they have. Compression of the aorta and vena cava occur during a tension pneumothorax, not a pericardial tamponade.

A man fell and landed directly on his right el bow when his arm was flexed. He is in moderate pain. There is obvious deformity to his elbow, his forearm is cool and pale, and you are unable to feel a radial pulse. What should you do? A) Make one attempt to realign the arm to restore distal circulation B) Start an IV line, administer analgesia, and then splint the injury C) Gently manipulate his arm until a pul e returns and then splint it D) Splint the injury in the position found and transport without delay

D) Splint the injury in the position found and transport without delay Feedback : A fractured or dislocated elbow-especially when associated with neurovascular compromise-is a true orthopedic emergency. In some cases, grossly deformed musculoskeletal injuries can be carefully realigned to facilitate splinting or restore distal circulation. This does not apply to elbow injuries! You should not manipulate a deformed elbow in the prehospital setting, even if signs of neurovascular compromise are present. The elbow is highly vascular and rich with nerves, and any manipulation may cause further damage (eg, severing an artery or nerve that was otherwi e compressed). Splint all elbow injuries in the position found and transport without delay. En route, notify the receiving facility so they can arrange for an orthopedic surgeon to be available. If the patient is in severe pain, administer analgesia (eg, fentanyl, morphine) as needed.

How do you dete rmine if the Achilles tendon is intact? A) Let the injured leg hang dependent and observe for the presence of a foot drop B) Have the patient dorsiflex the foot and ask about pain in the calf muscles C) Stroke the sole of the foot with a blunt object and observe for upward toe movement D) Squeeze the calf muscles of the injured leg and observe for plantar flexion of the foot

D) Squeeze the calf muscles of the injured leg and observe for plantar flexion of the foot Feedback : Rupture of the Achilles tendon usually occurs in athletes older than 30 years of age who are involved in start-and-stop sports such as basketball or football. The most immediate indications of Achilles tendon rupture are pain from the heel to the calf and a sudden inability for plantar flexion of the foot-extension of the ankle resulting in the forefoot moving away from the body. The Thompson test can be performed in the field to identify an Achilles tendon rupture. To perform this test, have the patient assum e a prone position and then squeeze the calf muscles of the injured leg. If the foot plantar flexes while squeezing die calf muscles, the tendon is likely intact. If there is no movement of the foot, the Ach illes tendon has likely been torn. If a patient experiences sharp calf muscle pain upon dorsiflexion of their foot, Homan's sign is said to be present; this finding indicates a deep venous thrombosis. A Babinski response is present if the patient's big toe moves upward when you stroke the sole of the foot with a blunt object. A present Babinski response is normal in infants; however, it is a sign of nervous system injury if it is present in older children and adults. The term "foot drop" is used to describe a neuromuscular disorder that affects the patient's ability to dorsiflex die foot. Foot drop is often caused by injury to the peroneal nerve-a division of the sciatic nerve that runs along the outside of the leg and bra nches off into each ankle, foot, and first two toes. It transmits signals to muscle groups responsible for ankle, foot, and toe movement and sensation.

Which of the following inj uries could cause obstructive shock? A) Pelvic fracture B) Aortic shearing C) Myocardial contusion D) Tension pneumothorax

D) Tension pneumothorax Feedback : The tenn "obstructive shock" refers to inadequate tissue perfusion secondary to any injury or condition that physically obstructs cardiopulmonary function. A pulmonary embolism, for example, causes obstructive shock because of a clot in a pulmonary artery that obstructs blood flow to the lungs for reoxygenation. A tension pneumothorax occurs when excessive air in the pleural space causes tension, which collapses the lung and then shifts to the contralateral (opposite) side of the chest. As pressure shifts across the mediastinum, venous return is impaired due to kinking of the vena cavae, and cardiac output is obstructed due to myocardial compression. Pericardia! tamponade i another injury that can result in obstructive shock. As t he pericardium fills with blood, the more pliable structures-namely the atria and vena cavae-become compressed, which reduces venous return to the heart and thereby di minishes stroke volume and cardiac output. Aortic shearing injuries cause profound hemorrhage, often to d1e point of exsanguination (bleeding to death). A fractured pelvis may also result in severe hemorrhage because the pelvis (like any bone) is vascular, femoral artery and/or vein injury may also occur. A myocard ial contusion, if severe enough, can result in impaired cardiac output, not because blood flow is obstructed, but because stroke volume is impaired due to cardiac damage.

Which of the fo llowing would l ikely occur if an unrestrained driver struck a solid object while traveling at 70 MPH? A) The majority of trauma will occur to the lower extremities and pelvis. B) The steering wheel or windshield will prevent ejection from the vehicle. C) The internal organs will collide wit h t he inside of the body at 140 MPH. D) The body will strike the interior of the vehicle while moving at 70 MPH.

D) The body will strike the interior of the vehicle while moving at 70 MPH. Feedback : D uring the first collision of a frontal (head-on) motor vehicle crash, the vehicle suddenly decelerates when it strikes a fixed object. During the second collision, the patient's body contin ues its forward movement at the same speed the vehicle was traveling until it strikes the inte,ior of the vehicle. During the third collision, the patient's internal organs continue their forward movement at the same speed the vehicle was traveling until they collide with the inside of the body. Unrestrained passengers are clearly at a higher risk of being ejected; they may strike the steering wheel or windshield before being ejected. Unre trained passengers take one of two pathways at the time of impact-up and over or down and under. The up and over padnvay often results in head and thoracic inju ries secondary to impact wi th the steering wheel or windshield, whereas the down and under pathway typically results in lower extremity and pelvic injuries as the knees collide wi th the dashboard.

A patie nt has a loss of proprioception following a spinal injury. What does this mean? A) The patient is unable to comprehend simple questions. B) The body's temperature assumes that of the environment. C) Motor function is decreased proximal to the site of the injury. D) The patient is unaware of one body part in relation to another.

D) The patient is unaware of one body part in relation to another. Feedback: Proprioception is the ability to sense the position, location, orientation, and movement of a part of the body in relation to another. M uscles, tendons. joints, and the inner ear contain proprioceptors, which relay positional information to the brain. The brain then analyzes this information and provides us with a sense of body orientation and movement. The inability of a patient to comprehend questions i s called receptive aphasia. A spinal cord injury results in compromised sensory and motor functions distal to the injury site, not proximal. The condition in which the body's temperature assume that of the environment is called poikilothermia.


Related study sets

4.04 Athletic Performance and Nutrition Needs

View Set

Parkland Formula Questions for BURNS Fluid Resuscitation with Rule of NINE

View Set

Testout 15.5.7 Practice Questions

View Set

Chapter 10 Replication of DNA and Chromosomes

View Set